Chapter 13 Probability

Short Answer Type Questions

1. For a loaded die, the probabilities of outcomes are given as under:

$P(1)=P(2)=0.2, P(3)=P(5)=P(6)=0.1$ and $P(4)=0.3$

The die is thrown, two times. Let $A$ and $B$ be the events, ‘same number each time’; and ‘a total score is 10 or more’; respectively. Determine whether or not A and B are independent.

Show Answer

Solution

A loaded die is thrown such that

$P(1)=P(2)=0.2, P(3)=P(5)=P(6)=0.1$ and $P(4)=0.3$ and die is thrown two times. Also given that:

$A=$ Same number each time and

$B=$ Total score is 10 or more.

$ \begin{aligned} & \text{ So, } P(A)=[P(1,1)+P(2,2)+P(3,3)+P(4,4)+P(5,5)+P(6,6)] \\ & =P(1) \cdot P(1)+P(2) \cdot P(2)+P(3) \cdot P(3)+P(4) \cdot P(4) \\ & +P(5) \cdot P(5)+P(6) \cdot P(6) \\ & =0.2 \times 0.2+0.2 \times 0.2+0.1 \times 0.1+0.3 \times 0.3+0.1 \times 0.1 \\ & +0.1 \times 0.1 \\ & =0.04+0.04+0.01+0.09+0.01+0.01=0.20 \end{aligned} $

Now $B=[(4,6),(6,4),(5,5),(5,6),(6,5),(6,6)]$

$P(B)=[P(4) \cdot P(6)+P(6) \cdot P(4)+P(5) \cdot P(5)+P(5) \cdot P(6)$ $+P(6) \cdot P(5)+P(6) \cdot P(6)$

$=0.3 \times 0.1+0.1 \times 0.3+0.1 \times 0.1+0.1 \times 0.1+0.1 \times 0.1$ $+0.1 \times 0.1$

$=0.03+0.03+0.01+0.01+0.01+0.01=0.10$

$A$ and $B$ both events will be independent if

$$ \begin{equation*} P(A \cap B)=P(A) \cdot P(B) \tag{i} \end{equation*} $$

Here,

$ (A \cap B)={(5,5),(6,6)} $

$ \therefore \quad P(A \cap B)=P(5,5)+P(6,6)=P(5) \cdot P(5)+P(6) \cdot P(6) $

$ =0.1 \times 0.1+0.1 \times 0.1=0.02 $

From eq. (i) we get

$ \begin{aligned} & 0.02=0.20 \times 0.10 \\ & 0.02=0.02 \end{aligned} $

Hence, $A$ and $B$ are independent events.

2. Refer to Exercise 1 above. If the die were fair, determine whether or not, the events A and B are independent.

Show Answer

Solution

According to the solution of Q. 1 , we have

$A={(1,1),(2,2),(3,3),(4,4),(5,5),(6,6)}$

$\therefore n(A)=6$ and $n(S)=6 \times 6=36$

So, $P(A)=\frac{n(A)}{n(S)}=\frac{6}{36}=\frac{1}{6}$

and $\quad B={(4,6),(6,4),(5,5),(5,6),(6,5),(6,6)}$

$n(B)=6$ and $n(S)=36$

$\therefore \quad P(B)=\frac{n(B)}{n(S)}=\frac{6}{36}=\frac{1}{6}$

$A \cap B={(5,5),(6,6)}$

$\therefore P(A \cap B)=\frac{2}{36}=\frac{1}{18}$

Therefore, if $A$ and $B$ are independent, then

$P(A \cap B)=P(A) \cdot P(B)$

$\Rightarrow \quad \frac{1}{18} \neq \frac{1}{6} \times \frac{1}{6} \Rightarrow \frac{1}{18} \neq \frac{1}{36}$

Hence, $A$ and $B$ are not independent events.

3. The probability that atleast one of the two events A and B occurs is 0.6 . If $A$ and $B$ occurs simultaneously with probability 0.3 , evaluate $P(\overline{A})+P(\overline{B})$.

Show Answer

Solution

We know that:

$A \cup B$ denotes that atleast one of the events occurs and $A \cap B$ denotes that the two events occur simultaneously.

$ \begin{matrix} \text{ So, } P(A \cup B) =P(A)+P(B)-P(A \cap B) \\ \Rightarrow 0.6 =P(A)+P(B)-0.3 \\ \Rightarrow 0.9 =P(A)+P(B) \\ \Rightarrow 0.9 =1-P(\overline{A})+1-P(\overline{B}) \\ \Rightarrow P(\overline{A})+P(\overline{B}) =2-0.9=1.1 \end{matrix} $

Hence, the required answer is 1.1.

4. A bag contains 5 red marbles and 3 black marbles. Three marbles are drawn one by one without replacement. What is the probability that atleast one of the three marbles drawn be black, if the first marble is red?

Show Answer

Solution

Let red marbles be represented with $R$ and black marble with B. The following three conditions are possible, if atleast one of the three marbles drawn be black and the first marble is red.

(i) $E_1$ : II ball is black and III is red

(ii) $E_2$ : II ball is black and III is also black

(iii) $E_3:$ II ball is red and III is black

$ \begin{aligned} & \therefore \quad P(E_1)=P(R_1) \cdot P(B_1 / R_1) \cdot P(R_2 / R_1 B_1)=\frac{5}{8} \cdot \frac{3}{7} \cdot \frac{4}{6}=\frac{60}{336}=\frac{5}{28} \\ & \quad P(E_2)=P(R_1) \cdot P(B_1 / R_1) \cdot P(B_2 / R_1 B_1)=\frac{5}{8} \cdot \frac{3}{7} \cdot \frac{2}{6}=\frac{30}{336}=\frac{5}{56} \\ & \text{ and } P(E_3)=P(R_1) \cdot P(R_2 / R_1) \cdot B(B_1 / R_1 R_2)=\frac{5}{8} \cdot \frac{4}{7} \cdot \frac{3}{6}=\frac{60}{336}=\frac{5}{28} \\ & \therefore \quad P(E)=P(E_1)+P(E_2)+P(E_3)=\frac{5}{28}+\frac{5}{56}+\frac{5}{28}=\frac{25}{56} \end{aligned} $

Hence the required probability is $\frac{25}{56}$.

5. Two dice are thrown together and the total score is noted. The events $E, F$ and $G$ are ‘a total of 4’ ‘a total of 9 or more’, and ‘a total divisible by $5^{\prime}$, respectively. Calculate $P(E), P(F)$ and $P(G)$ and decide which pairs of events, if any, are independent.

Show Answer

Solution

Two dice are thrown together

$\therefore n(S)=36$

$E=\text{ A total of } 4={(2,2),(1,3),(3,1)} \quad \therefore n(E)=3$

$F=\text{ A total of } 9 \text{ or more } $

$=\{(3,6),(6,3),(5,4),(4,5),(5,5),(4,6),(6,4),(5,6),(6,5),(6,6)\}$

$\therefore n(F)=10 ; G=\text{ A total divisible by } 5 $

$={(1,4),(4,1),(2,3),(3,2),(4,6),(6,4),(5,5)} \therefore n(G)=7$

Here, we see that $(E \cap F)=\phi$ and $(E \cap G)=\phi$

and $ (F \cap G)={(4,6),(6,4),(5,5)} $

$ \begin{aligned} \therefore \quad n(F & \cap G)=3 \text{ and }(E \cap F \cap G)=\phi \\ \therefore \quad P(E) & =\frac{n(E)}{n(S)}=\frac{3}{36}=\frac{1}{12} \\ P(F) & =\frac{n(F)}{n(S)}=\frac{10}{36}=\frac{5}{18} ; P(G)=\frac{n(G)}{n(S)}=\frac{7}{36} \\ P(F \cap G) & =\frac{3}{36}=\frac{1}{12} \text{ and } P(F) \cdot P(G)=\frac{5}{18} \cdot \frac{7}{36}=\frac{35}{648} \end{aligned} $

Since, $P(F \cap G) \neq P(F) . P(G)$

Hence, there is no pair of independent events.

6. Explain why the experiment of tossing a coin three times is said to have binomial distribution.

Show Answer

Solution

We know that random variable $X$ takes values $0,1,2,3, \ldots, n$ is said to be binomial distribution having parameters $n$ and $p$, if the probability is given by $P(X=r)={ }^{n} C_r p^{r} q^{n-r}$, where $q=1-p$ and $r=0,1,2,3, \ldots$

Similarly in case of tossing a coin 3 times,

$n=3$ and $X$ has the values $0,1,2,3$ with $p=\frac{1}{2}, q=\frac{1}{2}$.

Hence, it is said to have a binomial distribution.

7. $A$ and $B$ are two events such that $P(A)=\frac{1}{2}, P(B)=\frac{1}{3}$ and $P(A \cap B)=\frac{1}{4}$. Find:

(i) $P(A / B)$

(ii) $P(B / A)$

(iii) $P(A^{\prime} / B)$

(iv) $P(A^{\prime} / B^{\prime})$

Show Answer

Solution

We have $P(A)=\frac{1}{2}, P(B)=\frac{1}{3}$ and $P(A \cap B)=\frac{1}{4}$

$ \begin{aligned} P(A^{\prime}) & =1-\frac{1}{2}=\frac{1}{2}, P(B^{\prime})=1-\frac{1}{3}=\frac{2}{3} \\ P(A^{\prime} \cap B^{\prime}) & =1-P(A \cup B)=1-[P(A)+P(B)-P(A \cap B)] \\ & =1-[\frac{1}{2}+\frac{1}{3}-\frac{1}{4}]=1-[\frac{6+4-3}{12}]=1-\frac{7}{12}=\frac{5}{12} \end{aligned} $

(i) $P(A / B)=\frac{P(A \cap B)}{P(B)}=\frac{1 / 4}{1 / 3}=\frac{3}{4}$

(ii) $P(B / A)=\frac{P(A \cap B)}{P(A)}=\frac{1 / 4}{1 / 2}=\frac{1}{2}$

(iii) $P(A^{\prime} / B)=\frac{P(A^{\prime} \cap B)}{P(B)}=\frac{P(B)-P(A \cap B)}{P(B)}$

$ =1-\frac{P(A \cap B)}{P(B)}=1-\frac{1 / 4}{1 / 3}=1-\frac{3}{4}=\frac{1}{4} $

(iv) $P(A^{\prime} / B^{\prime})=\frac{P(A^{\prime} \cap B^{\prime})}{P(B^{\prime})}=\frac{5 / 12}{2 / 3}=\frac{5}{12} \times \frac{3}{2}=\frac{5}{8}$

8. Three events A, B and C have probabilities $\frac{2}{5}, \frac{1}{3}$ and $\frac{1}{2}$ respectively. Given that $P(A \cap C)=\frac{1}{5}$ and $P(B \cap C)=\frac{1}{4}$, find the values of $P(C / B)$ and $P(A^{\prime} \cap C^{\prime})$.

Show Answer

Solution

We have $P(A)=\frac{2}{5}, P(B)=\frac{1}{3}$ and $P(C)=\frac{1}{2}$

$ \begin{aligned} & P(A \cap C)=\frac{1}{5} \text{ and } P(B \cap C)=\frac{1}{4} \\ & \therefore \quad P(C / B)=\frac{P(B \cap C)}{P(B)}=\frac{1 / 4}{1 / 3}=\frac{3}{4} \\ & P(A^{\prime} \cap C^{\prime})=1-P(A \cup C) \\ & =1-[P(A)+P(C)-P(A \cap C)] \end{aligned} $

$ =1-[\frac{2}{5}+\frac{1}{2}-\frac{1}{5}]=1-\frac{7}{10}=\frac{3}{10} $

Hence, the required probabilities are $\frac{3}{4}$ and $\frac{3}{10}$.

9. Let $E_1$ and $E_2$ be two independent events such that $P(E_1)=p_1$ and $P(E_2)=p_2$. Describe in words of the events whose probabilities are:

(i) $p_1 p_2$

(ii) $(1-p_1) p_2$

(iii) $1-(1-p_1)(1-p_2)$

(iv) $p_1+p_2-2 p_1 p_2$

Show Answer

Solution

Here,

$ P(E_1)=p_1 \text{ and } P(E_2)=p_2 $

(i)

$ p_1 p_2=P(E_1) \cdot P(E_2)=P(E_1 \cap E_2) $

So, $E_1$ and $E_2$ occur.

(ii)

$ (1-p_1) \cdot p_2=P(E_1)^{\prime} \cdot P(E_2)=P(E_1^{\prime} \cap E_2) $

So, $E_1$ does not occur but $E_2$ occurs.

(iii) $1-(1-p_1)(1-p_2)=1-P(E_1)^{\prime} P(E_2)^{\prime}=1-P(E_1^{\prime} \cap E_2^{\prime})$

$ =1-[1-P(E_1 \cup E_2)]=P(E_1 \cup E_2) $

So, either $E_1$ or $E_2$ or both $E_1$ and $E_2$ occur.

(iv)

$ \begin{aligned} p_1+p_2-2 p_1 p_2 & =P(E_1)+P(E_2)-2 P(E_1) \cdot P(E_2) \\ & =P(E_1)+P(E_2)-2 P(E_1 \cap E_2) \\ & =P(E_1 \cup E_2)-2 P(E_1 \cap E_2) \end{aligned} $

So, either $E_1$ or $E_2$ occurs but not both.

10. A discrete random variable $X$ has the probability distribution given as below:

$\begin{array}{|l|l|l|l|l|} \hline X & 0.5 & 1 & 1.5 & 2 \\ \hline P(X) & k & k^2 & 2k^2 & k \\ \hline \end{array}$

(i) Find the value of $k$.

(ii) Determine the mean of the distribution.

Show Answer

Solution

For a probability distribution, we know that if $P_i \geq 0$

(i) $\sum _{i=1}^{n} P_i=1 \Rightarrow k+k^{2}+2 k^{2}+k=1$

$ \begin{aligned} & \Rightarrow \quad 3 k^{2}+2 k-1=0 \Rightarrow 3 k^{2}+3 k-k-1=0 \\ & \Rightarrow \quad 3 k(k+1)-1(k+1)=0 \Rightarrow(3 k-1)(k+1)=0 \\ & \therefore \quad k=\frac{1}{3} \text{ and } k=-1 \end{aligned} $

But $k \geq 0 \quad \therefore k=\frac{1}{3}$

(ii) Mean of the distribution

$ E(X)=\sum _{i=1}^{n} X_i P_i=0.5 k+1 . k^{2}+1.5(2 k^{2})+2 k $

11. Prove that

$ \begin{aligned} & =\frac{k}{2}+k^{2}+3 k^{2}+2 k=4 k^{2}+\frac{5}{2} k \\ & =4(\frac{1}{3})^{2}+\frac{5}{2}(\frac{1}{3})=\frac{4}{9}+\frac{5}{6}=\frac{23}{18} \end{aligned} $

(i) $P(A)=P(A \cap B)+P(A \cap \overline{B})$

(ii) $P(A \cup B)=P(A \cap B)+P(A \cap \overline{B})+P(\overline{A} \cap B)$

Show Answer

Solution

(i) To prove: $P(A)=P(A \cap B)+P(A \cap \overline{B})$ R.H.S. $=P(A \cap B)+P(A \cap \bar{{}B})$

$ \begin{aligned} & =P(A) \cdot P(B)+P(A) \cdot P(\overline{B})=P(A)[P(B)+P(\overline{B})] \\ & =P(A) \cdot 1 \\ & =P(A)=\text{ L.H.S. Hence proved. } \end{aligned} $

(ii) To prove: $P(A \cup B)=P(A \cap B)+P(A \cap \overline{B})+P(\overline{A} \cap B)$

R.H.S. $=P(A) \cdot P(B)+P(A) \cdot P(\bar{{}B})+P(\bar{{}A}) \cdot P(B)$

$ \begin{aligned} & =P(A) \cdot P(B)+P(A)[1-P(B)]+[1-P(A)] \cdot P(B) \\ & =P(A) \cdot P(B)+P(A)-P(A) \cdot P(B)+P(B)-P(A) \cdot P(B) \\ & =P(A)+P(B)-P(A \cap B) \\ & =P(A \cup B)=\text{ L.H.S. Hence proved. } \end{aligned} $

12. If $X$ is the number of tails in three tosses of a coin, determine the standard deviation of $X$.

Show Answer

Solution

Given that: $X=0,1,2,3$

$\therefore P(X=r)={ }^{n} C_r p^{r} q^{n-r}$

where $n=3, p=\frac{1}{2}, q=\frac{1}{2}$ and $r=0,1,2,3$

$P(X=0)=\frac{1}{2} \times \frac{1}{2} \times \frac{1}{2}=\frac{1}{8} ; P(X=1)=3 \times \frac{1}{2} \times \frac{1}{2} \times \frac{1}{2}=\frac{3}{8}$

$P(X=2)=3 \times \frac{1}{2} \times \frac{1}{2} \times \frac{1}{2}=\frac{3}{8} ; P(X=3)=\frac{1}{2} \times \frac{1}{2} \times \frac{1}{2}=\frac{1}{8}$

Probability distribution table is:

$\begin{array}{|l|l|l|l|l|} \hline X & 0 & 1 & 2 & 3 \\ \hline P(X) & \frac{1}{8} & \frac{3}{8} & \frac{3}{8} & \frac{1}{8} \\ \hline \end{array}$

$ \begin{aligned} & E(X)=0+1 \times \frac{3}{8}+2 \times \frac{3}{8}+3 \times \frac{1}{8}=\frac{3}{8}+\frac{6}{8}+\frac{3}{8}=\frac{12}{8}=\frac{3}{2} \\ & E(X^{2})=0+1 \times \frac{3}{8}+4 \times \frac{3}{8}+9 \times \frac{1}{8}=\frac{3}{8}+\frac{12}{8}+\frac{9}{8}=\frac{24}{8}=3 \end{aligned} $

We know that $Var(X)=E(X^{2})-[E(X)]^{2}=3-(\frac{3}{2})^{2}=3-\frac{9}{4}=\frac{3}{4}$

$\therefore$ Standard deviation $=\sqrt{Var(X)}=\sqrt{\frac{3}{4}}=\frac{\sqrt{3}}{2}$.

13. In a dice game, a player pays a stake of $Re 1$ for each throw of a die. She receives ₹ 5 if the die shows a 3, ₹ 2 , if the die shows a 1 or 6 , and nothing otherwise. What is the player’s expected profit per throw over a long series of throws?

Show Answer

Solution

Let $X$ be the random variable of profit per throw.

$\begin{array}{|l|l|l|l|l|} \hline X & -1 & 1 & 4 \\ \hline P(X) & \frac{1}{2} & \frac{1}{2} & \frac{1}{6} \\ \hline \end{array}$

Since, she loses ₹ 1 for getting any of 2, 4, 5 .

So, $P(X=-1)=\frac{1}{6}+\frac{1}{6}+\frac{1}{6}=\frac{3}{6}=\frac{1}{2}$

$ \begin{matrix} P(X=1)=\frac{1}{6}+\frac{1}{6}=\frac{2}{6}=\frac{1}{3} & (\because \text{ die showing } 1 \text{ or } 6) \\ P(X=4)=\frac{1}{6} & (\because \text{ die shows only a } 3) \end{matrix} $

Player’s expected profit $=\sum p_1 x_i$

$ =-1 \times \frac{1}{2}+1 \times \frac{1}{3}+4 \times \frac{1}{6}=-\frac{1}{2}+\frac{1}{3}+\frac{2}{3}=\frac{1}{2}=₹ 0.50 $

14. Three dice are thrown at the same time. Find the probability of getting three two’s, if it is known that the sum of the numbers on the dice was six.

Show Answer

Solution

The dice is thrown three times

$\therefore$ Sample space $n(S)=(6)^{3}=216$

Let $E_1$ be the event when the sum of numbers on the dice was six and $E_2$ be the event when three two’s occur.

$ \begin{aligned} \Rightarrow E_1=\{(1,1,4),(1,2,3),(1,3,2),(1,4,1),(2,1,3),(2,2,2),(2,3,1),(3,1,2),\\(3,2,1),(4,1,1)\}[\therefore E_2=\{2,2,2\}] \end{aligned} $

$\begin{aligned}\Rightarrow n(E_1)=10 \text{ and } n(E_2)=1\end{aligned}$

$\begin{aligned}\therefore P(E_2 / E_1)=\frac{P(E_1 \cap E_2)}{P(E_1)}=\frac{1 / 216}{10 / 216}=\frac{1}{10} . \end{aligned} $

15. Suppose 10,000 tickets are sold in a lottery each for Re. 1. First prize is of ₹ 3,000 and the second prize is of ₹ 2,000. There are three third prizes of ₹ 500 each. If you buy one ticket, what is your expectation?

Show Answer

Solution

Let $X$ be the random variable where $X=0,500,2000$ and 3000

$\begin{array}{|l|l|l|l|l|l|} \hline X & 0 & 500 & 2000 & 3000 \\ \hline P(X) & \frac{9995}{10000} & \frac{3}{10000} & \frac{1}{10000} & \frac{1}{10000} \\ \hline \end{array}$

Hence, expectation is ₹ 0.65 .

16. A bag contains 4 white and 5 black balls. Another bag contains 9 white and 7 black balls. A ball is transferred from the first bag to the second and then a ball is drawn at random from the second bag. Find the probability that the ball drawn is white.

Show Answer

Solution

Let $W_1$ and $W_2$ be two bags containing ( $4 W, 5 B)$ and (9 W, 7 B) balls respectively.

Let $E_1$ be the event that the

transferred ball from the bag

$W_1$ to $W_2$ is white and $E_2$ the event that the transferred ball is black.

And $E$ be the event that the ball drawn from the second bag is white.

$ \begin{aligned} \therefore P(E / E_1) & =\frac{10}{17}, \quad P(E / E_2)=\frac{9}{17} \\ P(E_1) & =\frac{4}{9} \text{ and } P(E_2)=\frac{5}{9} \\ \therefore \quad P(E) & =P(E_1) \cdot P(E / E_1)+P(E_2) \cdot P(E / E_2) \\ & =\frac{4}{9} \times \frac{10}{17}+\frac{5}{9} \times \frac{9}{17}=\frac{40}{153}+\frac{45}{153}=\frac{85}{153}=\frac{5}{9} \end{aligned} $

Hence, the required probability is $\frac{5}{9}$.

17. Bat I contains 3 black and 2 white balls, bag II contains 2 black and 4 white balls. A bag and a ball is selected at random. Determine the probability of selecting a black ball.

Show Answer

Solution

Given that $bag I={3 B, 2 W}$ and $\quad$ bag II $={2 B, 4 W}$

Let $E_1=$ The event that bag I is selected

$ E_2=\text{ The event that bag II is selected } $

and $E=$ The event that a black ball is selected

$ \begin{aligned} \therefore \quad P(E_1) & =\frac{1}{2}, P(E_2)=\frac{1}{2}, P(E _{/} E_1)=\frac{3}{5} \text{ and } P(E_2 E_2)=\frac{1}{3} \\ P(E) & =P(E_1) \cdot P(E / E_1)+P(E_2) \cdot P(E / E_2) \\ & =\frac{1}{2} \times \frac{3}{5}+\frac{1}{2} \times \frac{1}{3}=\frac{3}{10}+\frac{1}{6}=\frac{9+5}{30}=\frac{14}{30}=\frac{7}{15} \end{aligned} $

Hence, the required probability is $\frac{7}{15}$.

18. A box has 5 blue and 4 red balls. One ball is drawn at random and not replaced. Its colour is also not noted. Then another ball is drawn at random. What is the probability of second ball being blue?

Show Answer

Solution

Given that the box has 5 blue and 4 red balls.

Let $E_1$ be the event that first ball drawn is blue $E_2$ be the event that first ball drawn is red

and $E$ is the event that second ball drawn is blue.

$ \begin{aligned} \therefore \quad P(E) & =P(E_1) \cdot P(E / E_1)+P(E_2) \cdot P(E / E_2) \\ & =\frac{5}{9} \times \frac{4}{8}+\frac{4}{9} \times \frac{5}{8}=\frac{20}{72}+\frac{20}{72}=\frac{40}{72}=\frac{5}{9} \end{aligned} $

Hence, the required probability is $\frac{5}{9}$.

19. Four cards are successively drawn without replacement from a deck of 52 playing cards. What is the probability that all the four cards are Kings?

Show Answer

Solution

Let $E_1, E_2, E_3$ and $E_4$ be the events that first, second, third and fourth card is King respectively.

$\therefore P(E_1 \cap E_2 \cap E_3 \cap E_4)$

$=P(E_1) \cdot P(E_2 / E_1) \cdot P[\frac{E_3}{(E_1 \cap E_2)}] \cdot P[\frac{E_4}{(E_1 \cap E_2 \cap E_3 \cap E_4)}]$

$=\frac{4}{52} \times \frac{3}{51} \times \frac{2}{50} \times \frac{1}{49}=\frac{24}{52 \cdot 51 \cdot 50 \cdot 49}=\frac{1}{13 \cdot 17 \cdot 25 \cdot 49}=\frac{1}{27075}$

Hence, the required probability is $\frac{1}{27075}$.

20. A die is thrown 5 times. Find the probability that an odd number will come up exactly three times.

Show Answer

Solution

Here, $p=\frac{1}{6}+\frac{1}{6}+\frac{1}{6}=\frac{1}{2} \Rightarrow q=1-\frac{1}{2}=\frac{1}{2}$ and $n=5$

$ \begin{aligned} \therefore \quad P(x & =r)={ }^{n} C_r p^{r} q^{n-r} \\ & ={ }^{5} C_3(\frac{1}{2})^{3}(\frac{1}{2})^{5-3}=\frac{5 !}{3 ! 2 !} \cdot(\frac{1}{2})^{3} \cdot(\frac{1}{2})^{2}=10 \cdot \frac{1}{8} \cdot \frac{1}{4}=\frac{5}{16} \end{aligned} $

Hence, the required probability is $\frac{5}{16}$.

21. Ten coins are tossed. What is the probability of getting atleast 8 heads?

Show Answer

Solution

Here, $n=10, p=\frac{1}{2}, q=-\frac{1}{2}=\frac{1}{2}$

$ \begin{aligned} & P(X \geq 8)=P(x=8)+P(x=9)+P(x=10) \\ & ={ }^{10} C_8(\frac{1}{2})^{8}(\frac{1}{2})^{10-8}+{ }^{10} C_9(\frac{1}{2})^{9}(\frac{1}{2})^{10-9}+{ }^{10} C _{10}(\frac{1}{2})^{10}(\frac{1}{2})^{0} \\ & =\frac{10 !}{8 ! 2 !} \cdot(\frac{1}{2})^{8} \cdot(\frac{1}{2})^{2}+\frac{10 !}{9 ! 1 !}(\frac{1}{2})^{9}(\frac{1}{2})+(\frac{1}{2})^{10} \\ & =45 \cdot(\frac{1}{2})^{10}+10 \cdot(\frac{1}{2})^{10}+(\frac{1}{2})^{10}=(\frac{1}{2})^{10} \cdot(45+10+1) \\ & =56(\frac{1}{2})^{10}=56 \times \frac{1}{1024}=\frac{7}{128} \end{aligned} $

Hence, the required probability is $\frac{7}{128}$.

22. The probability of a man hitting a target is 0.25 . He shoots 7 times. What is the probability of his hitting at least twice?

Show Answer

Solution

Here $n=7, \quad p=0.25=\frac{25}{100}=\frac{1}{4} \quad$ and $q=1-\frac{1}{4}=\frac{3}{4}$

$ \begin{aligned} P(X \geq 2) & =1-[P(X=0)+P(X=1)] \\ & =1-[{ }^{7} C_0(\frac{1}{4})^{0}(\frac{3}{4})^{7}+{ }^{7} C_1(\frac{1}{4})^{1}(\frac{3}{4})^{6}] \\ & =1-[(\frac{3}{4})^{7}+\frac{7}{4}(\frac{3}{4})^{6}]=1-(\frac{3}{4})^{6}(\frac{3}{4}+\frac{7}{4}) \\ & =1-(\frac{3}{4})^{6}(\frac{10}{4})=1-\frac{729}{4096} \times \frac{10}{4}=1-\frac{7290}{16384} \\ & =\frac{16384-7290}{16384}=\frac{9094}{16384}=\frac{4547}{8192} \end{aligned} $

Hence, the required probability is $\frac{4547}{8192}$.

23. A lot of 100 watches is known to have 10 defective watches. If 8 watches are selected (one by one with replacement) at random, what is the probability that there will be atleast one defective watch?

Show Answer

Solution

Probability of defective watch out of 100 watches $=\frac{10}{100}=\frac{1}{10}$.

Here, $n=8, p=\frac{1}{10}$ and $q=1-\frac{1}{10}=\frac{9}{10}$ and $r \geq 1$

$P(X \geq 1)=1-P(x=0)=1-{ }^{8} C_0(\frac{1}{10})^{0}(\frac{9}{10})^{8-0}=1-(\frac{9}{10})^{8}$

Hence, the required probability is $1-(\frac{9}{10})^{8}$.

24. Consider the probability distribution of a random variable X:

$\begin{array}{|l|l|l|l|l|l|} \hline x & 0 & 1 & 2 & 3 & 4 \\ \hline y & 0.1 & 0.25 & 0.3 & 0.2 & 0.15 \\ \hline \end{array}$

Calculate:

(i) $V(\frac{X}{2})$

(ii) Variance of $X$.

Show Answer

Solution

Here, we have

$\begin{array}{|l|l|l|l|l|l|} \hline x & 0 & 1 & 2 & 3 & 4 \\ \hline y & 0.1 & 0.25 & 0.3 & 0.2 & 0.15 \\ \hline \end{array}$

$=E(X^{2})-[E(X)]^{2}$ |

where $E(X)=\sum _{i=1}^{n} x_i p_i$ and $E(X^{2})=\sum _{i=1}^{n} p_i x_i^{2}$

$\therefore \quad E(X)=0 \times 0.1+1 \times 0.25+2 \times 0.3+3 \times 0.2+4 \times 0.15$

$=0+0.25+0.6+0.6+0.6=2.05$

$E(X^{2})=0 \times 0.1+1 \times 0.25+4 \times 0.3+9 \times 0.2+16 \times 0.15$

$=0+0.25+1.2+1.8+2.40=5.65$

(i) $V(\frac{X}{2})=\frac{1}{4} V(X)=\frac{1}{4}[5.65-(2.05)^{2}]=\frac{1}{4}[5.65-4.2025]$

$=\frac{1}{4} \times 1.4475=0.361875$

$[\because V(\frac{X}{2})=\frac{1}{4} V(X)]$

(ii) $\quad Var(X)=1.4475$

25. The probability distribution of a random variable $X$ is given below:

$\begin{array}{|l|l|l|l|l|l|} \hline x & 0 & 1 & 2 & 3 \\ \hline y & k & \frac{k}{2} & \frac{k}{4} & \frac{k}{8} \\ \hline \end{array}$

(i) Determine the value of $k$.

(ii) Determine $P(X \leq 2)$ and $P(X>2)$

(iii) Find $P(X \leq 2)+P(X>2)$.

Show Answer

Solution

(i) We know that $P(0)+P(1)+P(2)+P(3)=1$

$ \begin{matrix} \Rightarrow k+\frac{k}{2}+\frac{k}{4}+\frac{k}{8} =1 \\ \Rightarrow \quad \frac{8 k+4 k+2 k+k}{8} =1 \Rightarrow 15 k=8 \\ \therefore \quad k =\frac{1}{15} \end{matrix} $

(ii)

$ \begin{aligned} P(X \leq 2) & =P(X=0)+P(X=1)+P(X=2) \\ & =k+\frac{k}{2}+\frac{k}{4}=\frac{7 k}{4}=\frac{7}{4} \times \frac{8}{15}=\frac{14}{15} \end{aligned} $

and $P(X>2)=P(X=3)=\frac{k}{8}=\frac{1}{8} \times \frac{8}{15}=\frac{1}{15}$

(iii) $P(X \leq 2)+P(X>2)=\frac{14}{15}+\frac{1}{15}=\frac{14+1}{15}=\frac{15}{15}=1$.

26. For the following probability distribution, determine standard deviation of the random variable $X$.

$\begin{array}{|l|l|l|l|} \hline x & 2 & 3 & 4 \\ \hline y & 0.2 & 0.5 & 0.3 \\ \hline \end{array}$

Show Answer

Solution

We know that: Standard deviation (S.D.) $=\sqrt{\text{ Variance }}$

$ \begin{aligned} & \therefore \quad Var(X)=E(X^{2})-[E(X)]^{2} \\ & E(X)=2 \times 0.2+3 \times 0.5+4 \times 0.3=0.4+1.5+1.2=3.1 \\ & E(X^{2})=4 \times 0.2+9 \times 0.5+16 \times 0.3=0.8+4.5+4.8=10.1 \\ & V(X)=10.1-(3.1)^{2}=10.1-9.61=0.49 \\ & \therefore \quad \text{ S.D. }=\sqrt{Var(X)}=\sqrt{0.49}=0.7 \end{aligned} $

27. A biased die is such that $P(4)=\frac{1}{10}$ and other scores being equally likely. The die is tossed twice. If $X$ is the ’number of fours seen’, find the variance of the random variable $X$.

Show Answer

Solution

Here, random variable $X=0,1,2$

$ \begin{aligned} P(4) & =\frac{1}{10}, P(\overline{4})=1-\frac{1}{10}=\frac{9}{10} \\ P(X=0) & =P(\overline{4}) \cdot P(\overline{4})=\frac{9}{10} \times \frac{9}{10}=\frac{81}{100} \\ P(X=1) & =P(\overline{4}) \cdot P(4)+P(4) \cdot P(\overline{4})=\frac{9}{10} \times \frac{1}{10}+\frac{1}{10} \times \frac{9}{10}=\frac{18}{100} \end{aligned} $

$P(X=2)=P(4) \cdot P(4)=\frac{1}{10} \times \frac{1}{10}=\frac{1}{100}$

$\begin{array}{|l|l|l|l|} \hline X & 0 & 1 & 2 \\ \hline P(X) & \frac{81}{100} & \frac{18}{100} & \frac{1}{100} \\ \hline \end{array}$

We know that $V(X)=E(X^{2})-[E(X)]^{2}$

$ \begin{aligned} E(X) & =0 \times \frac{81}{100}+1 \times \frac{18}{100}+\frac{2}{100}=\frac{20}{100}=\frac{1}{5} \\ E(X^{2}) & =0 \times \frac{81}{100}+1 \times \frac{18}{100}+4 \times \frac{1}{100}=\frac{22}{100}=\frac{11}{50} \\ \therefore \quad Var(X) & =\frac{11}{50}-(\frac{1}{5})^{2}=\frac{11}{50}-\frac{1}{25}=\frac{9}{50}=0.18 \end{aligned} $

Hence, the required variance $=0.18$.

28. A die is thrown three times. Let $X$ be the ’number of twos seen’, find the expectation of $X$.

Show Answer

Solution

Here, we have $X=0,1,2,3 \quad[\because$ die is thrown 3 times] and $p=\frac{1}{6}, q=\frac{5}{6}$

$\therefore P(X=0)=P(not 2) \cdot P(not 2) \cdot P(not 2)=\frac{5}{6} \cdot \frac{5}{6} \cdot \frac{5}{6}=\frac{125}{216}$

$P(X=1)=P(2) \cdot P(not 2) \cdot P(not 2)+P(not 2) \cdot P(2) \cdot P(not 2)$

$+P($ not 2$) . P(not 2) \cdot P(2)$

$=\frac{1}{6} \cdot \frac{5}{6} \cdot \frac{5}{6}+\frac{5}{6} \cdot \frac{1}{6} \cdot \frac{5}{6}+\frac{5}{6} \cdot \frac{5}{6} \cdot \frac{1}{6}=\frac{25}{216}+\frac{25}{216}+\frac{25}{216}=\frac{75}{216}$

$P(X=2)=P(2) \cdot P(2) \cdot P(not 2)+P(2) \cdot P(not 2) \cdot P(2)$

$+P($ not 2$) \cdot P(2) \cdot P(2)$

$=\frac{1}{6} \cdot \frac{1}{6} \cdot \frac{5}{6}+\frac{1}{6} \cdot \frac{5}{6} \cdot \frac{1}{6}+\frac{5}{6} \cdot \frac{1}{6} \cdot \frac{1}{6}=\frac{5}{216}+\frac{5}{216}+\frac{5}{216}=\frac{15}{216}$

$P(X=3)=P(2) \cdot P(2) \cdot P(2)=\frac{1}{6} \cdot \frac{1}{6} \cdot \frac{1}{6}=\frac{1}{216}$

Now $E(X)=\sum _{i=1}^{n} p_i x_i$

$ \begin{aligned} & i=1 \\ & 0 \times \frac{125}{216}+1 \times \frac{75}{216}+2 \times \frac{15}{216}+3 \times \frac{1}{216} \\ = & 0+\frac{75}{216}+\frac{30}{216}+\frac{3}{216}=\frac{75+30+3}{216}=\frac{108}{216}=\frac{1}{2} \end{aligned} $

Hence, the required expectation is $\frac{1}{2}$.

29. Two biased dice are thrown together. For the first die $P(6)=\frac{1}{2}$, the other scores being equally likely while for the second die, $P(1)=\frac{2}{5}$ and the other scores are equally likely. Find the probability distribution of ’the number of ones seen’.

Show Answer

Solution

Given that: for the first die, $P(6)=\frac{1}{2}$ and $P(\overline{6})=1-\frac{1}{2}=\frac{1}{2}$

$\Rightarrow P(1)+P(2)+P(3)+P(4)+P(5)=\frac{1}{2}$

But $P(1)=P(2)=P(3)=P(4)=P(5)$

$\therefore 5 . P(1)=\frac{1}{2} \Rightarrow P(1)=\frac{1}{10}$ and $P(\overline{1})=1-\frac{1}{10}=\frac{9}{10}$

For the second die, $P(1)=\frac{2}{5}$ and $P(\overline{1})=1-\frac{2}{5}=\frac{3}{5}$

Let $X$ be the number of one’s seen

$ \begin{aligned} \therefore X=0,1,2 \\ \Rightarrow \quad \begin{aligned} P(X=0) =P(\overline{1}) \cdot P _{II}(\overline{1})=\frac{9}{10} \cdot \frac{3}{5}=\frac{27}{50}=0.54 \\ P(X=1) =P(\overline{1}) \cdot P_I(1)+P(1) \cdot P _{(\overline{1})} \\ =\frac{9}{10} \cdot \frac{2}{5}+\frac{1}{10} \cdot \frac{3}{5}=\frac{18+3}{50}=\frac{21}{50}=0.42 \\ P(X=2) =\underset{I \hspace{2mm} II}{P(1) \cdot P(1)}=\frac{1}{10} \cdot \frac{2}{5}=\frac{2}{50}=0.04 \end{aligned} \end{aligned} $

Hence, the required probability distribution is

$\begin{array}{|l|l|l|l|} \hline X & 0 & 1 & 2 \\ \hline P(X) & 0.54 & 0.42 & 0.04 \\ \hline \end{array}$

30. Two probability distributions of the discrete random variable $X$ and $Y$ are given below.

$\begin{array}{|l|l|l|l|l|} \hline X & 0 & 1 & 2 & 3 \\ \hline P(X) & \frac{1}{5} & \frac{2}{5} & \frac{1}{5} & \frac{1}{5} \\ \hline \end{array}$

$\begin{array}{|l|l|l|l|l|} \hline (Y) & 0 & 1 & 2 & 3 \\ \hline P(Y) & \frac{1}{5} & \frac{3}{10} & \frac{2}{5} & \frac{1}{10} \\ \hline \end{array}$

Prove that: $E(Y^{2})=2 E(X)$.

Show Answer

Solution

First probability distribution is given by

$\begin{array}{|l|l|l|l|l|} \hline X & 0 & 1 & 2 & 3 \\ \hline P(X) & \frac{1}{5} & \frac{2}{5} & \frac{1}{5} & \frac{1}{5} \\ \hline \end{array}$

We know that, $E(X)=\sum _{i=1}^{n} P_i X_i$

$\Rightarrow E(X)=0 \cdot \frac{1}{5}+1 \cdot \frac{2}{5}+2 \cdot \frac{1}{5}+3 \cdot \frac{1}{5}=0+\frac{2}{5}+\frac{2}{5}+\frac{3}{5}=\frac{7}{5}$

For the second probability distribution,

$\begin{array}{|l|l|l|l|l|} \hline (Y) & 0 & 1 & 2 & 3 \\ \hline P(Y) & \frac{1}{5} & \frac{3}{10} & \frac{2}{5} & \frac{1}{10} \\ \hline \end{array}$

$ \begin{aligned} E(Y^{2}) & =0 \cdot \frac{1}{5}+1 \cdot \frac{3}{10}+4 \cdot \frac{2}{5}+9 \cdot \frac{1}{10} \\ & =0+\frac{3}{10}+\frac{8}{5}+\frac{9}{10}=\frac{28}{10}=\frac{14}{5} \end{aligned} $

Now $E(Y^{2})=\frac{14}{5}$ and $2 E(X)=2 \cdot \frac{7}{5}=\frac{14}{5}$

Hence, $E(Y^{2})=2 E(X)$.

31. A factory produces bulbs. The probability that any one bulb is defective is $\frac{1}{50}$ and they are packed in boxes of 10 . From the single box, find the probability that (i) none of the bulbs is defective (ii) exactly two bulbs are defective (iii) more than 8 bulbs work properly.

Show Answer

Solution

Let $X$ be the random variable denoting a bulb to be defective.

Here, $n=10, p=\frac{1}{50}, q=1-\frac{1}{50}=\frac{49}{50}$

We know that $P(X=r)={ }^{n} C_r p^{r} q^{n-r}$

(i) None of the bulbs is defective, i.e., $r=0$

$ P(x=0)={ }^{10} C_0(\frac{1}{50})^{0}(\frac{49}{50})^{10-0}=(\frac{49}{50})^{10} $

(ii) Exactly two bulbs are defective

$ \begin{aligned} \therefore \quad P(x=2) & ={ }^{10} C_2(\frac{1}{50})^{2}(\frac{49}{50})^{10-2} \\ & =45 \cdot \frac{(49)^{8}}{(50)^{10}}=45 \times(\frac{1}{50})^{10} \times(49)^{8} \end{aligned} $

(iii) More than 8 bulbs work properly

We can say that less than 2 bulbs are defective

$ P(x<2)=P(x=0)+P(x=1) $

$={ }^{10} C_0(\frac{1}{50})^{0}(\frac{49}{50})^{10}+{ }^{10} C_1(\frac{1}{50})^{1}(\frac{49}{50})^{9}=(\frac{49}{50})^{10}+\frac{1}{5}(\frac{49}{50})^{9}$

$ =(\frac{49}{50})^{9}(\frac{49}{50}+\frac{1}{5})=(\frac{49}{50})^{9}(\frac{59}{50})=\frac{59(49)^{9}}{(50)^{10}} . $

32. Suppose you have two coins which appear identical in your pocket. You know that one is fair and one is 2-headed coin. If you take one out, toss it and get a head, what is the probability that it was a fair coin?

Show Answer

Solution

Let $E_1=$ Event that the coin is fair

$ E_2=\text{ Event that the coin is 2-headed } $

and $H=$ Event that the tossed coin gets head.

$ P(E_1)=\frac{1}{2}, \quad P(E_2)=\frac{1}{2}, \quad P(H / E_1)=\frac{1}{2}, \quad P(H / E_2)=1 $

$\therefore$ Using Bayes’ Theorem, we get

$ \begin{aligned} P(E_1 / H) & =\frac{P(E_1) \cdot P(H / E_1)}{P(E_1) \cdot P(H / E_1)+P(E_2) \cdot P(H / E_2)} \\ & =\frac{\frac{1}{2} \cdot \frac{1}{2}}{\frac{1}{2} \cdot \frac{1}{2}+\frac{1}{2} \cdot 1}=\frac{\frac{1}{4}}{\frac{1}{4}+\frac{1}{2}}=\frac{\frac{1}{3}}{\frac{3}{4}}=\frac{1}{3} \end{aligned} $

Hence the required probability is $\frac{1}{3}$.

33. Suppose that 6% of the people with blood group O are left handed and $10 %$ of those with other blood groups are left handed, $30 %$ of the people have blood group O. If a left handed person is selected at random, what is the probability that he/she will have blood group $O$ ?

Show Answer

Solution

Let $E_1=$ The event that a person selected is of blood group $O$ $E_2=$ The event that the person selected is of other group and $H=$ The event that selected person is left handed

$ \begin{matrix} \therefore \quad P(E_1) & =0.30 \text{ and } & P(E_2)=0.70 \\ P(H / E_1) & =0.06 & P(H / E_2)=0.10 \end{matrix} $

So, from Bayes’ Theorem

$ \begin{aligned} & \quad P(\frac{E_1}{H})=\frac{P(E_1) \cdot P(H / E_1)}{P(E_1) \cdot P(H / E_1)+P(E_2) \cdot P(H / E_2)} \\ & =\frac{0.30 \times 0.06}{0.30 \times 0.06+0.70 \times 0.10}=\frac{0.018}{0.018+0.070}=\frac{0.018}{0.088}=\frac{9}{44} \\ & \text{ Hence, the required probability is } \frac{9}{44} . \end{aligned} $

34. Two natural numbers $r$ and $s$ are drawn one at a time, without replacement from the set $S={1,2,3, \ldots, n}$. Find $P(r \leq p / s \leq p)$, where $p \in S$.

Show Answer

Solution

Given that: $S={1,2,3, \ldots, n}$

$ \therefore \quad P(r \leq p / s \leq p)=\frac{P(P \cap S)}{P(S)}=\frac{p-1}{n} \times \frac{n}{n-1}=\frac{p-1}{n-1} $

Hence, the required probability is $\frac{p-1}{n-1}$.

35. Find the probability distribution of the maximum of the two scores obtained when a die is thrown twice. Determine also the mean of the distribution.

Show Answer

Solution

Let $X$ be the random variable scores when a die is thrown twice.

and

$ \begin{aligned} X & =1,2,3,4,5,6 \\ S & =\{(1,1),(1,2),(2,1),(2,2),(1,3),(2,3),(3,1),(3,2),(3,3),(3,4),(3,5),..,(6,6)\} \end{aligned} $

So, $\quad P(X=1)=\frac{1}{6} \cdot \frac{1}{6}=\frac{1}{36}$

$ \begin{aligned} & P(X=2)=\frac{1}{6} \cdot \frac{1}{6}+\frac{1}{6} \cdot \frac{1}{6}+\frac{1}{6} \cdot \frac{1}{6}=\frac{3}{36} \\ & P(X=3)=\frac{1}{6} \cdot \frac{1}{6}+\frac{1}{6} \cdot \frac{1}{6}+\frac{1}{6} \cdot \frac{1}{6}+\frac{1}{6} \cdot \frac{1}{6}+\frac{1}{6} \cdot \frac{1}{6}=\frac{5}{36} \end{aligned} $

Similarly

$ P(X=4)=\frac{7}{36} ; P(X=5)=\frac{9}{36} \text{ and } P(X=6)=\frac{11}{36} $

So, the required distribution is

$\begin{array}{|l|l|l|l|l|l|l|} \hline X & 1 & 2 & 3 & 4 & 5 & 6 \\ \hline P(X) & \frac{1}{36} & \frac{3}{36} & \frac{5}{36} & \frac{7}{36} & \frac{9}{36} &\frac{11}{36} \\ \hline \end{array}$

Now, the mean $E(X)=\sum _{i=1}^{n} x_i p_i$

$ \begin{aligned} & =1 \times \frac{1}{36}+2 \times \frac{3}{36}+3 \times \frac{5}{36}+4 \times \frac{7}{36}+5 \times \frac{9}{36}+6 \times \frac{11}{36} \\ & =\frac{1}{36}+\frac{6}{36}+\frac{15}{36}+\frac{28}{36}+\frac{45}{36}+\frac{66}{36}=\frac{161}{36} \end{aligned} $

Hence, the required mean $=\frac{161}{36}$.

36. The random variable $X$ can take only the values $0,1,2$. If $P(X=0)=P(X=1)=p$ and $E(X^{2})=E(X)$, then find the value of $p$.

Show Answer

Solution

Given that: $\quad X=0,1,2$ and $P(X)$ at $X=0$ and 1 is $p$. Let $P(X)$ at $X=2$ is $x$

$\Rightarrow \quad p+p+x=1 \quad \Rightarrow \quad x=1-2 p$

Now we have the following distributions.

$\begin{array}{|l|l|l|l|} \hline X & 0 & 1 & 2 \\ \hline P(X) & p & p & 1-2p \\ \hline \end{array}$

$\therefore \quad E(X)=0 . p+1 . p+2(1-2 p)=p+2-4 p=2-3 p$

and $\quad E(X^{2})=0 . p+1 . p+4(1-2 p)=p+4-8 p=4-7 p$

Given that: $\quad E(X^{2})=E(X)$

$\therefore \quad 4-7 p=2-3 p \Rightarrow 4 p=2 \Rightarrow p=\frac{1}{2}$

Hence, the required value of $p$ is $\frac{1}{2}$.

37. Find the variance of the distribution:

$\begin{array}{|l|l|l|l|l|l|l|} \hline X & 0 & 1 & 2 & 3 & 4 & 5 \\ \hline P(X) & \frac{1}{6} & \frac{5}{18} & \frac{2}{9} & \frac{1}{6} & \frac{1}{9} &\frac{1}{18} \\ \hline \end{array}$

Show Answer

Solution

We know that:

$ \begin{aligned} & \text{ Variance }(X)=E(X^{2})-[E(X)]^{2} \\ & E(X)=\sum _{i=1}^{n} p_i x_i \\ &=0 \times \frac{1}{6}+1 \times \frac{5}{18}+2 \times \frac{2}{9}+3 \times \frac{1}{6}+4 \times \frac{1}{9}+5 \times \frac{1}{18} \\ &=0+\frac{5}{18}+\frac{4}{9}+\frac{3}{6}+\frac{4}{9}+\frac{5}{18}=\frac{5+8+9+8+5}{18}=\frac{35}{18} \\ & E(X^{2})=0 \times \frac{1}{6}+1 \times \frac{5}{18}+4 \times \frac{2}{9}+9 \times \frac{1}{6}+16 \times \frac{1}{9}+25 \times \frac{1}{18} \\ &=\frac{5}{18}+\frac{8}{9}+\frac{9}{6}+\frac{16}{9}+\frac{25}{18}=\frac{5+16+27+32+25}{18}=\frac{105}{18} \\ & \therefore \text{ Var }(X)=\frac{105}{18}-\frac{35}{18} \times \frac{35}{18}=\frac{1890-1225}{324}=\frac{665}{324} \\ & \text{ Hence, the required variance is } \frac{665}{324} . \end{aligned} $

38. A and B throw a pair of dice alternately. A wins the game if he gets a total of 6 and $B$ wins if she gets a total of 7. If A starts the game, find the probability of winning the game by $A$ in third throw of the pair of dice.

Show Answer

Solution

Let $A_1$ be the event of getting a total of 6

$ ={(2,4),(4,2),(1,5),(5,1),(3,3)} $

and $B_1$ be the event of getting a total of 7

$ ={(2,5),(5,2),(1,6),(6,1),(3,4),(4,3)} $

Let $P(A_1)$ is the probability, if $A$ wins in a throw $=\frac{5}{36}$

and $P(B_1)$ is the probability, if $B$ wins in a throw $=\frac{1}{6}$

$\therefore$ The required probability of winning $A$ in his third throw

$ =P(\overline{A}_1) \cdot P(\overline{B}_1) \cdot P(A_1)=\frac{31}{36} \cdot \frac{5}{6} \cdot \frac{5}{36}=\frac{775}{7776} . $

39. Two dice are tossed. Find whether the following two events $A$ and $B$ are independent. $A={(x, y): x+y=11}$ and $B={(x, y): x \neq 5}$ where $(x, y)$ denotes a typical sample point.

Show Answer

Solution

Given that:

$A=\{(x, y): x+y=11\} \text{ and } B=\{(x, y): x \neq 5\}$

$\therefore \quad A=\{(5,6),(6,5)\},$

$B=\{(1,1),(1,2),(1,3),(1,4),(1,5),(1,6),(2,1),(2,2),(2,3),(2,4),(2,5),(2,6),(3,1),$ $(3,2),(3,3),(3,4),(3,5),(3,6),(4,1),(4,2),(4,3),(4,4),(4,5),(4,6),(6,1),(6,2),$ $ (6,3),(6,4),(6,5),(6,6)\}$

$ \begin{aligned} & \Rightarrow n(A)=2, n(B)=30 \text{ and } n(A \cap B)=1 \\ & \therefore \quad P(A)=\frac{2}{36}=\frac{1}{18} \text{ and } P(B)=\frac{30}{36}=\frac{5}{6} \\ & \Rightarrow \quad P(A) \cdot P(B)=\frac{1}{18} \cdot \frac{5}{6}=\frac{5}{108} \text{ and } P(A \cap B)=\frac{1}{36} \end{aligned} $

Since $P(A).P(B) \neq P(P \cap B)$

Hence, A and B are not independent.

40. An urn contains $m$ white and $n$ black balls. A ball is drawn at random and is put back into the urn along with $k$ additional balls of the same colour as that of the ball drawn. A ball is again drawn at random. Show that the probability of drawing a white ball now does not depend on $k$.

Show Answer

Solution

Let $A$ be the event having $m$ white and $n$ black balls

$ \begin{aligned} & E_1={\text{ first ball drawn of white colour }} \\ & E_2={\text{ first ball drawn of black colour }} \end{aligned} $

$ \begin{aligned} & E_3={\text{ second ball drawn of white colour }} \\ & \therefore \quad P(E_1)=\frac{m}{m+n} \text{ and } P(E_2)=\frac{n}{m+n} \\ & P(E_3 / E_1)=\frac{m+k}{m+n+k} \text{ and } P(E_3 / E_2)=\frac{m}{m+n+k} \\ & \text{ Now } P(E_3)=P(E_1) \cdot P(E_3 / E_1)+P(E_2) \cdot P(E_3 / E_2) \\ &=\frac{m}{m+n} \times \frac{m+k}{m+n+k}+\frac{n}{m+n} \times \frac{m}{m+n+k} \\ &=\frac{m}{m+n+k}[\frac{m+k}{m+n}+\frac{n}{m+n}] \\ &=\frac{m}{m+n+k}[\frac{m+n+k}{m+n}]=\frac{m}{m+n} \end{aligned} $

Hence, the probability of drawing a white ball does not depend upon $k$.

Long Answer Type Questions

41. Three bags contain a number of red and white balls as follows, Bag I: 3 red balls, Bag II: 2 red balls and 1 white ball and Bag III: 3 white balls. The probability that bag $i$ will be chosen and a ball is selected from it is $\frac{i}{6}$, where $i=1,2,3$.

What is the probability that (i) a red ball will be selected (ii) a white ball is selected?

Show Answer

Solution

Given that:

Bag I: 3 red balls and no white ball

Bag II: 2 red balls and 1 white ball

Bag III: no red ball and 3 white balls

Let $E_1, E_2$ and $E_3$ be the events of choosing Bag I, Bag II and Bag III respectively and a ball is drawn from it.

$\therefore P(E_1)=\frac{1}{6}, P(E_2)=\frac{2}{6}$ and $P(E_3)=\frac{3}{6}$

(i) Let $E$ be the event that red ball is selected

$ \begin{aligned} \therefore P(E) & =P(E_1) \cdot P(E / E_1)+P(E_2) \cdot P(E / E_2)+P(E_3) \cdot P(E / E_3) \\ & =\frac{1}{6} \cdot \frac{3}{3}+\frac{2}{6} \cdot \frac{2}{3}+\frac{3}{6} \cdot 0=\frac{3}{18}+\frac{4}{18}=\frac{7}{18} \end{aligned} $

(ii) Let $F$ be the event that a white ball is selected

$ \begin{matrix} \therefore P(F) & =1-P(E) & {[P(E)+P(F)=1]} \\ & =1-\frac{7}{18}=\frac{11}{18} & \end{matrix} $

Hence, the required probabilities are $\frac{7}{18}$ and $\frac{11}{18}$.

42. Refer to Exercise Q. 41 above. If a white ball is selected, what is the probability that it came from (i) Bag II (ii) Bag III?

Show Answer

Solution

Referring to Exercise Q.41, we will use here Bayes’ Theorem

$ \begin{aligned} & \text{ (i) } \begin{aligned} P(E_2 / F) & =\frac{P(E_2) \cdot P(F / E_2)}{P(E_1) \cdot P(F / E_1)+P(E_2) \cdot P(F / E_2)+P(E_3) \cdot P(F / E_3)} \\ & =\frac{\frac{2}{6} \cdot \frac{1}{3}}{\frac{1}{6} \cdot 0+\frac{2}{6} \cdot \frac{1}{3}+\frac{3}{6} \cdot 1}=\frac{\frac{2}{18}}{\frac{2}{18}+\frac{3}{6}}=\frac{2}{11} \\ \text{ (ii) } P(E_3 / F) & =\frac{P(E_3) \cdot P(F / E_3)}{P(E_1) \cdot P(F / E_1)+P(E_2) \cdot P(F / E_2)+P(E_3) \cdot P(F / E_3)} \\ & =\frac{\frac{3}{6} \cdot 1}{\frac{1}{6} \cdot 0+\frac{2}{6} \cdot \frac{1}{3}+\frac{3}{6} \cdot 1}=\frac{\frac{3}{6}}{\frac{2}{18}+\frac{3}{6}}=\frac{3}{6} \times \frac{18}{11}=\frac{9}{11} \end{aligned} \\ & \text{ Hence, the required probabilities are } \frac{2}{11} \text{ and } \frac{9}{11} . \end{aligned} $

43. A shopkeeper sells three types of flower seeds $A_1, A_2$ and $A_3$. They are sold as a mixture, where the proportions are $4: 4: 2$, respectively. The germination rates of the three types of seeds are $45 %, 60 %$ and $35 %$. Calculate the probability

(i) of a randomly chosen seed to germinate

(ii) that it will not germinate given that the seed is of type $A_3$

(iii) that it is of the type $A_2$ given that a randomly chosen seed does not germinate.

Show Answer

Solution

Given that $A_1: A_2: A_3=4: 4: 2$

$\therefore P(A_1)=\frac{4}{10}, P(A_2)=\frac{4}{10}$ and $P(A_3)=\frac{2}{10}$

where $A_1, A_2$ and $A_3$ are the three types of seeds.

Let $E$ be the event that a seed germinates and $\overline{E}$ be the event that a seed does not germinate

$\therefore P(\frac{E}{A_1})=\frac{45}{100}, P(\frac{E}{A_2})=\frac{60}{100}$ and $P(\frac{E}{A_3})=\frac{35}{100}$

and $P(\frac{\overline{E}}{A_1})=\frac{55}{100}, P(\frac{\overline{E}}{A_2})=\frac{40}{100}$ and $P(\frac{\overline{E}}{A_3})=\frac{65}{100}$ (i)

$ \begin{aligned} P(E) & =P(A_1) \cdot P(\frac{E}{A_1})+P(A_2) \cdot P(\frac{E}{A_2})+P(A_3) \cdot P(\frac{E}{A_3}) \\ & =\frac{4}{10} \cdot \frac{45}{100}+\frac{4}{10} \cdot \frac{60}{100}+\frac{2}{10} \cdot \frac{35}{100} \\ & =\frac{180}{1000}+\frac{240}{1000}+\frac{70}{1000}=\frac{490}{1000}=0.49 \end{aligned} $

(ii) $P(\overline{E} / A_3)=1-P(E / A_3)=1-\frac{35}{100}=\frac{65}{100}=0.65$

(iii) Using Bayes’ Theorem, we get

$ \begin{aligned} & P(A_2 / \overline{E})=\frac{P(A_2) \cdot P(\overline{E} / A_2)}{P(A_1) \cdot P(\overline{E} / A_1)+P(A_2) \cdot P(\overline{E} / A_2)+P(A_3) \cdot P(\overline{E} / A_3)} \\ &= \frac{\frac{4}{10} \cdot \frac{40}{100}}{\frac{4}{10} \cdot \frac{55}{100}+\frac{4}{10} \cdot \frac{40}{100}+\frac{2}{10} \cdot \frac{65}{100}} \\ &=\frac{\frac{160}{1000}}{\frac{220}{1000}+\frac{160}{1000}+\frac{130}{1000}}=\frac{160}{220+160+130}=\frac{160}{510}=\frac{16}{51}=0.314 \end{aligned} $

Hence, the required probability is $\frac{16}{51}$ or 0.314 .

44. A letter is known to have come either from TATA NAGAR or from CALCUTTA. On the envelope, just two consecutive letters TA are visible. What is the probability that the letter came from TATA NAGAR?

Show Answer

Solution

Let $E_1$ : The event that the letter comes from TATANAGAR and $E_2$ : The event that the letter comes from CALCUTTA Also $E_3$ : The event that on the letter, two consecutive letters TA are visible

$\therefore P(E_1)=\frac{1}{2}$ and $P(E_2)=\frac{1}{2}$ and $P(\frac{E_3}{E_1})=\frac{2}{8}$ and $P(\frac{E_3}{E_2})=\frac{1}{7}$

$[\because$ For TATA NAGAR, the two consecutive letters visible are TA, AT, TA, AN, NA, AG, GA, AR] $\therefore P(E_3 / E_1)=\frac{2}{8}$ and [For CALCUTTA, the two consecutive letters visible are $CA, AL, LC, CU, UT, TT$ and TA] So, $P(E_3 / E_2)=\frac{1}{7}$ Now using Bayes’ Theorem, we have

$ P(E_1 / E_3)=\frac{P(E_1) \cdot P(E_3 / E_1)}{P(E_1) \cdot P(E_3 / E_1)+P(E_2) \cdot P(E_3 / E_2)} $

$ =\frac{\frac{1}{2} \cdot \frac{2}{8}}{\frac{1}{2} \cdot \frac{2}{8}+\frac{1}{2} \cdot \frac{1}{7}}=\frac{\frac{1}{8}}{\frac{1}{8}+\frac{1}{14}}=\frac{\frac{1}{8}}{\frac{7+4}{56}}=\frac{7}{11} $

Hence, the required probability is $\frac{7}{11}$.

45. There are two bags, one of which contains 3 black and 4 white balls while the other contains 4 black and 3 white balls. A die is thrown. If it shows up 1 or 3 , a ball is taken from the first bag but if it shows up any other number, a ball is chosen from the second bag. Find the probability of choosing a black ball.

Show Answer

Solution

Let $E_1$ be the event of selecting Bag I and $E_2$ be the event of selecting Bag II

Let $E_3$ be the event that black ball is selected

$ \begin{aligned} \therefore \quad P(E_1) & =\frac{2}{6}=\frac{1}{3} \text{ and } P(E_2)=1-\frac{1}{3}=\frac{2}{3} \\ P(E_3 / E_1) & =\frac{3}{7} \text{ and } P(E_3 / E_2)=\frac{4}{7} \\ \therefore \quad P(E_3) & =P(E_1) \cdot P(E_3 / E_1)+P(E_2) \cdot P(E_3 / E_2) \\ & =\frac{1}{3} \cdot \frac{3}{7}+\frac{2}{3} \cdot \frac{4}{7}=\frac{3+8}{21}=\frac{11}{21} \end{aligned} $

Hence, the required probability is $\frac{11}{21}$.

46. There are three urns containing 2 white and 3 black balls, 3 white and 2 black balls and 4 white and 1 black balls, respectively. There is an equal probability of each urn being chosen. A ball is drawn at random from the chosen urn and it is found to be white. Find the probability that the ball drawn was from the second urn.

Show Answer

Solution

We have 3 urns:

$\therefore$ Probabilities of choosing either of the urns are

$ P(U_1)=P(U_2)=P(U_3)=\frac{1}{3} $

Let $H$ be the event of drawing white ball from the chosen urn.

$ \begin{aligned} & \therefore P(H / U_1)=\frac{2}{5}, P(H / U_2)=\frac{3}{5} \text{ and } P(H / U_3)=\frac{4}{5} \\ & \therefore P(U_2 / H)=\frac{P(U_2) \cdot P(H / U_2)}{P(U_1) \cdot P(H / U_1)+P(U_2) \cdot P(H / U_2)+P(U_3) \cdot P(H / U_3)} \\ & \quad=\frac{\frac{1}{3} \cdot \frac{3}{5}}{\frac{1}{3} \cdot \frac{2}{5}+\frac{1}{3} \cdot \frac{3}{5}+\frac{1}{3} \cdot \frac{4}{5}}=\frac{\frac{3}{5}}{\frac{2}{5}+\frac{3}{5}+\frac{4}{5}}=\frac{3}{9}=\frac{1}{3} \end{aligned} $

Hence, the required probability is $\frac{1}{3}$.

47. By examining the chest $X$-ray, the probability that $TB$ is detected when a person is actually suffering is 0.99 . The probability of an healthy person diagnosed to have TB is 0.001 . In a certain city, 1 in 1000 people suffer from TB. A person is selected at random and is diagnosed to have TB. What is the probability that he actually has TB?

Show Answer

Solution

Let $E_1$ : Event that a person has TB

$ E_2 \text{ : Event that a person does not have TB } $

and $H$ : Event that the person is diagnosed to have TB.

$ \begin{aligned} & \therefore \quad P(E_1)=\frac{1}{1000}=0.001, P(E_2)=1-\frac{1}{1000}=\frac{999}{1000}=0.999 \\ & P(H / E_1)=0.99, P(H / E_2)=0.001 \\ & \therefore P(E_1 / H)=\frac{P(E_1) \cdot P(H / E_1)}{P(E_1) \cdot P(H / E_1)+P(E_2) \cdot P(H / E_2)} \\ & =\frac{0.001 \times 0.99}{0.001 \times 0.99+0.999 \times 0.001}=\frac{0.99}{0.99+0.999} \\ & =\frac{0.990}{0.990+0.999}=\frac{990}{1989}=\frac{110}{221} \end{aligned} $

Hence, the required probability is $\frac{110}{221}$.

48. An item is manufactured by three machines A, B and C. Out of the total number of items manufactured during a specified period, $50 %$ are manufactured on machine A, $30 %$ on B and $20 %$ on C. $2 %$ of items produced on A and $2 %$ of items produced on B are defective and 3% of these produced on machine $C$ are defective. All the items are stored at one godown. One item is drawn at random and is found to be defective. What is the probability that it was manufactured on machine A?

Show Answer

Solution

Let $E_1$ : The event that the item is manufactured on machine A

$E_2$ : The event that the item is manufactured on machine $B$

$E_3$ : The event that the item is manufactured on machine C

Let $H$ be the event that the selected item is defective.

$\therefore$ Using Bayes’ Theorem,

$ \begin{aligned} & P(E_1)=\frac{50}{100}, P(E_2)=\frac{30}{100}, P(E_3)=\frac{20}{100} \\ & P(H / E_1)=\frac{2}{100}, P(H / E_2)=\frac{2}{100} \text{ and } P(H / E_3)=\frac{3}{100} \\ & \therefore P(E_1 / H)=\frac{P(E_1) \cdot P(H / E_1)}{P(E_1) \cdot P(H / E_1)+P(E_2) \cdot P(H / E_2)+P(E_3) \cdot P(H / E_3)} \\ & =\frac{\frac{50}{100} \times \frac{2}{100}}{\frac{50}{100} \times \frac{2}{100}+\frac{30}{100} \times \frac{2}{100}+\frac{20}{100} \times \frac{3}{100}} \\ & =\frac{100}{100+60+60}=\frac{100}{220}=\frac{10}{22}=\frac{5}{11} \end{aligned} $

49. Let $X$ be a discrete random variable whose probability distribution is defined as follows:

$ P(X=x)=\begin{matrix} k(x+1) & \text{ for } x=1,2,3,4 \\ 2 k x & \text{ for } x=5,6,7 \\ 0, & \text{ otherwise } \end{matrix} . $

where $k$ is a constant-Calculate:

(i) the value of $k$ (ii) $E(X)$ (iii) Standard deviation of $X$.

Show Answer

Solution

(i) Here, $P(X=x)=k(x+1)$ for $x=1,2,3,4$

So, $P(X=1)=k(1+1)=2 k ; P(X=2)=k(2+1)=3 k$

$ P(X=3)=k(3+1)=4 k ; P(X=4)=k(4+1)=5 k $

Also, $P(X=x)=2 k x$ for $x=5,6,7$

$ \begin{aligned} & P(X=5)=2(5) k=10 k ; P(X=6)=2(6) k=12 k \\ & P(X=7)=2(7) k=14 k \end{aligned} $

and for otherwise it is 0 .

$\therefore$ The probability distribution is given by

$\begin{array}{|l|l|l|l|l|l|l|l|l|} \hline X & 1 & 2 & 3 & 4 & 5 & 6 & 7 & \text{otherwise} \\ \hline P(X) & 2k & 3k & 4k & 5k & 10k & 12k & 14k & 0 \\ \hline \end{array}$

$ \begin{aligned} & \text{ We know that } \sum _{i=1}^{n} P(X_i)=1 \\ & \text{ So, } 2 k+3 k+4 k+5 k+10 k+12 k+14 k=1 \\ & \Rightarrow \quad 50 k=1 \Rightarrow k=\frac{1}{50} \\ & \text{ Hence, the value of } k \text{ is } \frac{1}{50} \end{aligned} $

(ii) Now the probability distribution is

$\begin{array}{|l|l|l|l|l|l|l|l|} \hline X & 1 & 2 & 3 & 4 & 5 & 6 & 7 \\ \hline P(X) & \frac{2}{50} & \frac{3}{50} & \frac{4}{50} & \frac{5}{50} & \frac{10}{50} & \frac{12}{50} &\frac{14}{50} \\ \hline \end{array}$

$ \begin{aligned} E(X)=1 \times \frac{2}{50}+2 \times \frac{3}{50}+3 \times \frac{4}{50}+4 \times \frac{5}{50}+5 \times \frac{10}{50}+ & 6 \times \frac{12}{50} \\ & +7 \times \frac{14}{50} \end{aligned} $

$ =\frac{2}{50}+\frac{6}{50}+\frac{12}{50}+\frac{20}{50}+\frac{50}{50}+\frac{72}{50}+\frac{98}{50}=\frac{260}{50}=\frac{26}{5}=5.2 $

(iii) We know that Standard deviation (SD) $=\sqrt{\text{ Variance }}$ Variance $=E(X^{2})-[E(X)]^{2}$

$ \begin{aligned} & \begin{aligned} & E(X^{2})= 1 \times \frac{2}{50}+4 \times \frac{3}{50}+9 \times \frac{4}{50}+16 \times \frac{5}{50}+25 \times \frac{10}{50} \\ &=\frac{2}{50}+\frac{12}{50}+\frac{36}{50}+\frac{80}{50}+\frac{250}{50}+\frac{432}{50}+\frac{686}{50}=\frac{1498}{50} \end{aligned} \\ & \therefore \text{ Variance }(X)=\frac{1498}{50}-(\frac{26}{5})^{2} \\ & =\frac{1498}{50}-\frac{676}{25}=\frac{1498-1352}{50}=\frac{146}{50}=2.92 \\ & \therefore \quad \text{ S.D }=\sqrt{2.92}=1.7 \text{ (approx.) } \end{aligned} $

50. The probability distribution of a discrete random variable $X$ is given as under:

$\begin{array}{|l|l|l|l|l|l|l|} \hline X & 1 & 2 & 4 & 2A & 3A & 5A \\ \hline P(X) & \frac{1}{2} & \frac{1}{5} & \frac{3}{25} & \frac{1}{10} & \frac{1}{25} & \frac{1}{25} \\ \hline \end{array}$

Calculate: (i) The value of $A$ if $E(X)=2.94$; (ii) Variance of $X$

Show Answer

Solution

(i) We know that: $E(X)=\sum _{i=1}^{n} P_i X_i$

$ \begin{matrix} \therefore & E(X) & =1 \times \frac{1}{2}+2 \times \frac{1}{5}+4 \times \frac{3}{25}+2 A \times \frac{1}{10}+3 A \times \frac{1}{25}+5 A \times \frac{1}{25} \\ & 2.94 & =\frac{1}{2}+\frac{2}{5}+\frac{12}{25}+\frac{A}{5}+\frac{3 A}{25}+\frac{A}{5} \\ \Rightarrow & 2.94 & =0.5+0.4+0.48+\frac{13 A}{25}=1.38+\frac{13 A}{25} \\ \Rightarrow & 2.94 & -1.38=\frac{13 A}{25} \Rightarrow 1.56=\frac{13 A}{25} \\ \Rightarrow \quad & A & =\frac{1.56 \times 25}{13}=0.12 \times 25 \\ & \therefore \quad & A & =3 \end{matrix} $

(ii) Now the distribution becomes

$\begin{array}{|l|l|l|l|l|l|l|} \hline X & 1 & 2 & 4 & 6 & 9 & 15 \\ \hline P(X) & \frac{1}{2} & \frac{1}{5} & \frac{3}{25} & \frac{1}{10} & \frac{1}{25} & \frac{1}{25} \\ \hline \end{array}$

$E(X^{2})=1 \times \frac{1}{2}+4 \times \frac{1}{5}+16 \times \frac{3}{25}+36 \times \frac{1}{10}+81 \times \frac{1}{25}+225 \times \frac{1}{25}$

$ \begin{aligned} & =\frac{1}{2}+\frac{4}{5}+\frac{48}{25}+\frac{36}{10}+\frac{81}{25}+\frac{225}{25} \\ & =0.5+0.8+1.92+3.6+3.24+9.00=19.06 \end{aligned} $

Variance $(X)=E(X^{2})-[E(X)]^{2}$

$ =19.06-(2.94)^{2}=19.06-8.64=10.42 $

51. The probability distribution of a random variable $X$ is given as under:

where $k$ is a constant. Calculate:

$ P(X=x)=\begin{cases} k x^{2} & \text{ for } x=1,2,3 \\ 2 k x & \text{ for } x=4,5,6 \\ 0 & \text{ otherwise } \end{cases} . $

(i) $E(X)$ (ii) $E(3 X^{2})$ (iii) $P(X \geq 4)$

Show Answer

Solution

Given that: $\quad P(X=x)=\begin{cases} k x^{2} & \text{ for } x=1,2,3 \\ 2 k x & \text{ for } x=4,5,6 \\ 0 & \text{ otherwise } \end{cases} .$

$\therefore$ Probability distribution of random variable $X$ is

$\begin{array}{|l|l|l|l|l|l|l|l|l|} \hline X & 1 & 2 & 3 & 4 & 5 & 6 & \text{otherwise} \\ \hline P(X) & k & 4k & 9k & 8k & 10k & 0 \\ \hline \end{array}$

We know that $\sum _{i=1}^{n} P(X_i)=1$

$\therefore k+4 k+9 k+8 k+10 k+12 k=1 \Rightarrow 44 k=1 \Rightarrow k=\frac{1}{44}$

(i) $E(X)=\sum _{i=1}^{n} P_i X_i=1 \times k+2 \times 4 k+3 \times 9 k+4 \times 8 k+5 \times 10 k+6 \times 12 k$ $=k+8 k+27 k+32 k+50 k+72 k=190 k$ $=190 \times \frac{1}{44}=\frac{95}{22}=4.32$ (approx.)

(ii) $E(3 X^{2})=3[k+4 \times 4 k+9 \times 9 k+16 \times 8 k+25 \times 10 k+36 \times 12 k]$

$=3[k+16 k+81 k+128 k+250 k+432 k]=3[908 k]$

$=3 \times 908 \times \frac{1}{44}=\frac{2724}{44}=61.9$ (approx.)

(iii) $P(X \geq 4)=P(X=4)+P(X=5)+P(X=6)$

$=8 k+10 k+12 k=30 k$

$=30 \times \frac{1}{44}=\frac{15}{22}$.

52. A bag contains $(2 n+1)$ coins. It is known that $n$ of these coins have a head on both sides whereas the rest of the coins are fair. A coin is picked up at random from the bag and is tossed. If the probability that the toss results in a head is $\frac{31}{42}$, determine the value of $n$.

Show Answer

Solution

Given that $n$ coins are two headed coins and the remaining $(n+1)$ coins are fair.

Let $E_1$ : the event that unfair coin is selected

$E_2$ : the event that the fair coin is selected

$E$ : the event that the toss results in a head

$\therefore \quad P(E_1)=\frac{n}{2 n+1}$ and $P(E_2)=\frac{n+1}{2 n+1}$

$P(E / E_1)=1$ (sure event) and $P(E / E_2)=\frac{1}{2}$

$\therefore \quad P(E)=P(E_1) \cdot P(E / E_1)+P(E_2) \cdot P(E / E_2)$

$=\frac{n}{2 n+1} \cdot 1+\frac{n+1}{2 n+1} \cdot \frac{1}{2}=\frac{1}{2 n+1}(n+\frac{n+1}{2})$

$=\frac{1}{2 n+1}(\frac{2 n+n+1}{2})=\frac{3 n+1}{2(2 n+1)}$

But $\quad P(E)=\frac{31}{42}$ (given)

$ \begin{matrix} \therefore \frac{3 n+1}{2(2 n+1)}=\frac{31}{42} & \Rightarrow & \frac{3 n+1}{2 n+1} & =\frac{31}{21} \\ & \Rightarrow & 63 n+21 & =62 n+31 \\ & \Rightarrow & & n & =10 \end{matrix} $

Hence, the required value of $n$ is 10 .

53. Two cards are drawn successively without replacement from a well shuffled deck of cards. Find the mean and standard deviation of the random variable $X$ where $X$ is the number of aces.

Show Answer

Solution

Let $X$ be the random variable such that $X=0,1,2$

and $E=$ the event of drawing an ace

and $F=$ the event of drawing non-ace.

$\therefore P(E)=\frac{4}{52}$ and $P(\overline{E})=\frac{48}{52}$

Now $P(X=0)=P(\overline{E}) \cdot P(\overline{E})=\frac{48}{52} \cdot \frac{47}{51}=\frac{188}{221}$

$ \begin{gathered} P(X=1)=P(E) \cdot P(\bar{{}E})+P(\bar{{}E}) \cdot P(E)=\frac{4}{52} \times \frac{48}{51}+\frac{48}{52} \times \frac{4}{51}=\frac{32}{221} \\ P(X=2)=P(E) \cdot P(E)=\frac{4}{52} \cdot \frac{3}{51}=\frac{1}{221} \end{gathered} $

We have Distribution Table:

$\begin{array}{|l|l|l|l|} \hline X & 0 & 1 & 2 \\ \hline P(X) & \frac{188}{221} & \frac{32}{221} & \frac{1}{221} \\ \hline \end{array}$

Now, Mean $E(X)=0 \times \frac{188}{221}+1 \times \frac{32}{221}+2 \times \frac{1}{221}=\frac{32}{221}+\frac{2}{221}=\frac{34}{221}=\frac{2}{13}$

$ E(X^{2})=0 \times \frac{188}{221}+1 \times \frac{32}{221}+4 \times \frac{1}{221}=\frac{32}{221}+\frac{4}{221}=\frac{36}{221} $

$\therefore \quad$ Variance $=E(X^{2})-[E(X)]^{2}$

$ =\frac{36}{221}-(\frac{2}{13})^{2}=\frac{36}{221} \quad \frac{4}{169}=\frac{468-68}{13 \times 221}=\frac{400}{2873} $

Standard deviation $=\sqrt{\frac{400}{2873}}=0.377$ (approx.)

54. A die is tossed twice. A ‘success’ is getting an even number on a toss. Find the variance of the number of successes.

Show Answer

Solution

Let $E$ be the event of getting even number on tossing a die.

$\therefore P(E)=\frac{3}{6}=\frac{1}{2}$ and $P(\overline{E})=1-\frac{1}{2}=\frac{1}{2}$

Here $X=0,1,2$

$ \begin{gathered} P(X=0)=P(\bar{{}E}) \cdot P(\bar{{}E})=\frac{1}{2} \cdot \frac{1}{2}=\frac{1}{4} \\ P(X=1)=P(E) \cdot P(\bar{{}E})+P(\bar{{}E}) \cdot P(E)=\frac{1}{2} \cdot \frac{1}{2}+\frac{1}{2} \cdot \frac{1}{2}=\frac{1}{4}+\frac{1}{4}=\frac{2}{4} \\ P(X=2)=P(E) \cdot P(E)=\frac{1}{2} \cdot \frac{1}{2}=\frac{1}{4} \end{gathered} $

$\therefore$ Probability distribution table is

$\begin{array}{|l|l|l|l|} \hline X & 0 & 1 & 2 \\ \hline P(X) & \frac{1}{4} & \frac{2}{4} & \frac{1}{4} \\ \hline \end{array}$

$ \begin{aligned} E(X) & =0 \times \frac{1}{4}+1 \times \frac{2}{4}+2 \times \frac{1}{4}=\frac{2}{4}+\frac{2}{4}=1 \\ E(X^{2}) & =0 \times \frac{1}{4}+1 \times \frac{2}{4}+4 \times \frac{1}{4}=\frac{3}{2} \end{aligned} $

$\therefore$ Variance $(X)=E(X^{2})-[E(X)]^{2}=\frac{3}{2}-1=\frac{1}{2}=0.5$

55. There are 5 cards numbered 1 to 5 , one number on one card. Two cards are drawn at random without replacement. Let $X$ denotes the sum of the numbers on two cards drawn. Find the mean and variance of $X$.

Show Answer

Solution

Here, sample space $S=\{(1,2),(2,1),(1,3),(3,1),(2,3),(3,2)$, $(1,4),(4,1),(1,5),(5,1),(2,4),(4,2),(2,5),(5,2),(3,4),(4,3)$, $(3,5),(5,3),(5,4),(4,5)\}$

$\therefore n(S)=20$

Let $X$ be the random variable denoting the sum of the numbers on two cards drawn.

$ \therefore \quad X=3,4,5,6,7,8,9 $

So, $\quad P(X=3)=\frac{2}{20}$

$ \begin{aligned} & P(X=4)=\frac{2}{20} \\ & P(X=6)=\frac{4}{20} \\ & P(X=8)=\frac{2}{20} \end{aligned} $

$ \begin{aligned} & P(X=5)=\frac{4}{20} \\ & P(X=7)=\frac{4}{20} \end{aligned} $

$ \begin{matrix} P(X=9)=\frac{2}{20} \\ \therefore \text{ Mean, } E(X)=3 \times \frac{2}{20}+4 \times \frac{2}{20}+5 \times \frac{4}{20}+6 \times \frac{4}{20}+7 \times \frac{4}{20} \\ +8 \times \frac{2}{20}+9 \times \frac{2}{20} \\ =\frac{6}{20}+\frac{8}{20}+\frac{20}{20}+\frac{24}{20}+\frac{28}{20}+\frac{16}{20}+\frac{18}{20}=\frac{120}{20}=6 \\ E(X^{2})=9 \times \frac{2}{20}+16 \times \frac{2}{20}+25 \times \frac{4}{20}+36 \times \frac{4}{20}+49 \times \frac{4}{20}+64 \times \frac{2}{20}+81 \times \frac{2}{20} \\ =\frac{18}{20}+\frac{32}{20}+\frac{100}{20}+\frac{144}{20}+\frac{196}{20}+\frac{128}{20}+\frac{162}{20}=\frac{780}{20}=39 \\ \therefore \text{ Variance }(X)=E(X^{2})-[E(X)]^{2}=39-(6)^{2}=39-36=3 \end{matrix} $

Objective Type Questions

56. If $P(A)=\frac{4}{5}$ and $P(A \cap B)=\frac{7}{10}$, then $P(B / A)$ is equal to

(a) $\frac{1}{10}$

(b) $\frac{1}{8}$

(c) $\frac{7}{8}$

(d) $\frac{17}{20}$

Show Answer

Solution

Given that: $P(A)=\frac{4}{5}$ and $P(A \cap B)=\frac{7}{10}$

$ \therefore \quad P(B / A)=\frac{P(A \cap B)}{P(A)}=\frac{7 / 10}{4 / 5}=\frac{7}{8} $

Hence, the correct option is (c).

57. If $P(A \cap B)=\frac{7}{10}$ and $P(B)=\frac{17}{20}$, then $P(A / B)$ equals

(a) $\frac{14}{17}$

(b) $\frac{17}{20}$

(c) $\frac{7}{8}$

(d) $\frac{1}{8}$

Show Answer

Solution

Given that: $P(A \cap B)=\frac{7}{10}$ and $P(B)=\frac{17}{20}$

$ \therefore \quad P(A / B)=\frac{P(A \cap B)}{P(B)}=\frac{7 / 10}{17 / 20}=\frac{14}{17} $

Hence, the correct option is $(a)$.

58. If $P(A)=\frac{3}{10}, P(B)=\frac{2}{5}$ and $P(A \cup B)=\frac{3}{5}$, then

$P(B / A)+P(A / B)$ equals to

(a) $\frac{1}{4}$

(b) $\frac{1}{3}$

(c) $\frac{5}{12}$

(d) $\frac{7}{12}$

Show Answer

Solution

Here, $P(A)=\frac{3}{10}, P(B)=\frac{2}{5}$ and $P(A \cup B)=\frac{3}{5}$

$ \begin{aligned} & P(A \cup B)=P(A)+P(B)-P(A \cap B) \\ & \Rightarrow \quad \frac{3}{5}=\frac{3}{10}+\frac{2}{5}-P(A \cap B) \\ & \Rightarrow \quad P(A \cap B)=\frac{3}{10}+\frac{2}{5}-\frac{3}{5}=\frac{3+4-6}{10}=\frac{1}{10} \\ & \text{ Now } P(A / B)+P(B / A)=\frac{P(A \cap B)}{P(B)}+\frac{P(A \cap B)}{P(A)} \\ & =\frac{1 / 10}{2 / 5}+\frac{1 / 10}{3 / 10}=\frac{1}{4}+\frac{1}{3}=\frac{7}{12} \end{aligned} $

Hence, the correct option is $(d)$.

59. If $P(A)=\frac{2}{5}, P(B)=\frac{3}{10}$ and $P(A \cap B)=\frac{1}{5}$, then $P(A^{\prime} / B^{\prime}) \cdot P(B^{\prime} / A^{\prime})$ is equal to

(a) $\frac{5}{6}$

(b) $\frac{5}{7}$

(c) $\frac{25}{42}$

(d) 1

Show Answer

Solution

Given that: $P(A)=\frac{2}{5}, P(B)=\frac{3}{10}$ and $P(A \cap B)=\frac{1}{5}$

$ \begin{aligned} & P(A^{\prime})=1-\frac{2}{5}=\frac{3}{5}, P(B^{\prime})=1-\frac{3}{10}=\frac{7}{10} \\ & \text{ and } P(A^{\prime} \cap B^{\prime})=1-P(A \cup B)=1-[P(A)+P(B)-P(A \cap B)] \\ &=1-[\frac{2}{5}+\frac{3}{10}-\frac{1}{5}]=1-[\frac{1}{5}+\frac{3}{10}]=1-\frac{5}{10}=\frac{1}{2} \\ & \therefore \quad P(A^{\prime} / B^{\prime})=\frac{P(A^{\prime} \cap B^{\prime})}{P(B^{\prime})}=\frac{1 / 2}{7 / 10}=\frac{5}{7} \\ & \text{ and } P(B^{\prime} / A^{\prime})=\frac{P(A^{\prime} \cap B^{\prime})}{P(A^{\prime})}=\frac{1 / 2}{3 / 5}=\frac{5}{6} \\ & \therefore P(A^{\prime} / B^{\prime}) \cdot P(B^{\prime} / A^{\prime})=\frac{5}{7} \times \frac{5}{6}=\frac{25}{42} \end{aligned} $

Hence, the correct option is (c).

60. If $A$ and $B$ are two events such that $P(A)=\frac{1}{2}, P(B)=\frac{1}{3}$ and $P(A / B)=\frac{1}{4}$, then $P(A^{\prime} \cap B^{\prime})$ equals

(a) $\frac{1}{12}$

(b) $\frac{3}{4}$

(c) $\frac{1}{4}$

(d) $\frac{3}{16}$

Show Answer

Solution

Given that: $P(A)=\frac{1}{2}, P(B)=\frac{1}{3}$ and $P(A / B)=\frac{1}{4}$

$ \begin{aligned} P(A / B) & =\frac{P(A \cap B)}{P(B)} \\ \frac{1}{4} & =\frac{P(A \cap B)}{1 / 3} \Rightarrow P(A \cap B)=\frac{1}{4} \times \frac{1}{3}=\frac{1}{12} \end{aligned} $

Now $\quad P(A^{\prime} \cap B^{\prime})=1-P(A \cup B)$

$ =1-[P(A)+P(B)-P(A \cap B)] $

$=1-[\frac{1}{2}+\frac{1}{3}-\frac{1}{12}]=1-[\frac{5}{6}-\frac{1}{12}]=1-\frac{9}{12}=\frac{3}{12}=\frac{1}{4}$

Hence, the correct option is (c).

61. If $P(A)=0.4, P(B)=0.8$ and $P(B / A)=0.6$ then $P(A \cup B)$ is equal to

(a) 0.24

(b) 0.3

(c) 0.48

(d) 0.96

Show Answer

Solution

Given that: $P(A)=0.4, P(B)=0.8$ and $P(B / A)=0.6$

$ \begin{aligned} P(B / A)=\frac{P(A \cap B)}{P(A)} & \Rightarrow 0.6=\frac{P(A \cap B)}{0.4} \\ \therefore \quad P(A \cap B) & =0.6 \times 0.4=0.24 \\ P(A \cup B) & =P(A)+P(B)-P(A \cap B) \\ & =0.4+0.8-0.24=1.20-0.24=0.96 \end{aligned} $

Hence, the correct option is (d).

62. If $A$ and $B$ are two events and $A \neq \phi, B \neq \phi$, then

(a) $P(A / B)=P(A) \cdot P(B)$

(b) $P(A / B)=\frac{P(A \cap B)}{P(B)}$

(c) $P(A / B) \cdot P(B / A)=1$

(d) $P(A / B)=\frac{P(A)}{P(B)}$

Show Answer

Solution

Given that: $A=\phi$ and $B \neq \phi$,

then

$ P(A / B)=\frac{P(A \cap B)}{P(B)} $

Hence, the correct option is (b).

63. A and $B$ are events such that $P(A)=0.4, P(B)=0.3$ and $P(A \cup B)$ $=0.5$. Then $P(B^{\prime} \cap A)$ equals

(a) $\frac{2}{3}$

(b) $\frac{1}{2}$

(c) $\frac{3}{10}$

(d) $\frac{1}{5}$

Show Answer

Solution

Given that: $P(A)=0.4, P(B)=0.3$ and $P(A \cup B)=0.5$

$ \begin{aligned} P(A \cup B) & =P(A)+P(B)-P(A \cap B) \\ 0.5 & =0.4+0.3-P(A \cap B) \\ P(A \cap B) & =0.4+0.3-0.5=0.2 \\ \therefore \quad P(B^{\prime} \cap A) & =P(A)-P(A \cap B) \\ & =0.4-0.2=0.2=\frac{1}{5} \end{aligned} $

Hence, the correct option is $(d)$.

64. If $A$ and $B$ are two events such that $P(B)=\frac{3}{5}, P(A / B)=\frac{1}{2}$ and $P(A \cup B)=\frac{4}{5}$, then $P(A)$ equals

(a) $\frac{3}{10}$

(b) $\frac{1}{5}$

(c) $\frac{1}{2}$

(d) $\frac{3}{5}$

Show Answer

Solution

Given that: $P(B)=\frac{3}{5}, P(A / B)=\frac{1}{2}$ and $P(A \cup B)=\frac{4}{5}$

We know that $P(A / B)=\frac{P(A \cap B)}{P(B)} \Rightarrow \frac{1}{2}=\frac{P(A \cap B)}{3 / 5}$

$\therefore \quad P(A \cap B)=\frac{3}{10}$

Now $\quad P(A \cup B)=P(A)+P(B)-P(A \cap B)$

$ \begin{aligned} \frac{4}{5} & =P(A)+\frac{3}{5}-\frac{3}{10} \\ \Rightarrow \quad P(A) & =\frac{4}{5}-\frac{3}{5}+\frac{3}{10}=\frac{1}{5}+\frac{3}{10}=\frac{5}{10}=\frac{1}{2} \end{aligned} $

Hence, the correct option is (c).

65. In Exercise 64 above, $P(B / A^{\prime})$ is equal to

(a) $\frac{1}{5}$

(b) $\frac{3}{10}$

(c) $\frac{1}{2}$

(d) $\frac{3}{5}$

Show Answer

Solution

According to Exercise 64, we have

$ \begin{aligned} & P(B)=\frac{3}{5}, P(A / B)=\frac{1}{2}, P(A \cup B)=\frac{4}{5} \\ & P(B / A^{\prime})=\frac{P(B \cap A^{\prime})}{P(A^{\prime})}=\frac{P(B)-P(A \cap B)}{1-P(A)}=\frac{\frac{3}{5}-\frac{3}{10}}{1-\frac{1}{2}}=\frac{\frac{3}{10}}{\frac{1}{2}}=\frac{3}{5} \end{aligned} $

Hence, the correct option is $(d)$.

66. If $P(B)=\frac{3}{5}, P(A / B)=\frac{1}{2}$ and $P(A \cup B)=\frac{4}{5}$, then

$P(A \cup B)^{\prime}+P(A^{\prime} \cup B)=$

(a) $\frac{1}{5}$

(b) $\frac{4}{5}$

(c) $\frac{1}{2}$

(d) 1

Show Answer

Solution

Given that: $P(B)=\frac{3}{5}, P(A / B)=\frac{1}{2}$ and $P(A \cup B)=\frac{4}{5}$

$ \begin{aligned} P(A / B) & =\frac{P(A \cap B)}{P(B)} \\ \Rightarrow \quad \frac{1}{2} & =\frac{P(A \cap B)}{3 / 5} \Rightarrow P(A \cap B)=\frac{3}{10} \\ P(A \cup B) & =P(A)+P(B)-P(A \cap B) \\ \frac{4}{5} & =P(A)+\frac{3}{5}-\frac{3}{10} \\ \therefore \quad P(A) & =\frac{4}{5}-\frac{3}{5}+\frac{3}{10}=\frac{1}{5}+\frac{3}{10}=\frac{5}{10}=\frac{1}{2} \end{aligned} $

Now $P(A \cup B)^{\prime}+P(A^{\prime} \cup B)$

$ \begin{aligned} & =1-P(A \cup B)+1-P(A \cap B^{\prime}) \\ & =2-\frac{4}{5}-P(A) \cdot P(B^{\prime}) \\ & =\frac{6}{5}-\frac{1}{2} \cdot(1-\frac{3}{5})=\frac{6}{5}-\frac{1}{2} \times \frac{2}{5}=\frac{6}{5}-\frac{1}{5}=\frac{5}{5}=1 \end{aligned} $

Hence, the correct option is $(d)$.

67. Let $P(A)=\frac{7}{13}, P(B)=\frac{9}{13}$ and $P(A \cap B)=\frac{4}{13}$. Then $P(A^{\prime} / B)$ is equal to

(a) $\frac{6}{13}$

(b) $\frac{4}{13}$

(c) $\frac{4}{9}$

(d) $\frac{5}{9}$

Show Answer

Solution

Given that: $P(A)=\frac{7}{13}, P(B)=\frac{9}{13}$ and $P(A \cap B)=\frac{4}{13}$

$ P(A^{\prime} / B)=\frac{P(A^{\prime} \cap B)}{P(B)}=\frac{P(B)-P(A \cap B)}{P(B)}=\frac{\frac{9}{13}-\frac{4}{13}}{\frac{9}{13}}=\frac{\frac{5}{13}}{\frac{9}{13}}=\frac{5}{9} $

Hence, the correct option is $(d)$.

68. If $A$ and $B$ are such events that $P(A)>0$ and $P(B) \neq 1$ then $P(A^{\prime} / B^{\prime})$ equals

(a) $1-P(A / B)$

(b) $1-P(A^{\prime} / B)$

(c) $\frac{1-P(A \cup B)}{P(B^{\prime})}$

(d) $\frac{P(A^{\prime})}{P(B^{\prime})}$

Show Answer

Solution

Given that: $P(A)>0$ and $P(B) \neq 1$

$ \therefore \quad P(A^{\prime} / B^{\prime})=\frac{P(A^{\prime} \cap B^{\prime})}{P(B^{\prime})}=\frac{1-P(A \cup B)}{P(B^{\prime})} $

Hence, the correct option is (c).

69. If $A$ and $B$ are two independent events with $P(A)=\frac{3}{5}$ and $P(B)=\frac{4}{9}$, then $P(A^{\prime} \cap B^{\prime})$ equals

(a) $\frac{4}{15}$

(b) $\frac{8}{45}$

(c) $\frac{1}{3}$

(d) $\frac{2}{9}$

Show Answer

Solution

Given that: A and B are independent events

such that

$ \begin{aligned} & P(A)=\frac{3}{5} \quad \therefore P(A^{\prime})=1-\frac{3}{5}=\frac{2}{5} \\ & P(B)=\frac{4}{9} \quad \therefore P(B^{\prime})=1-\frac{4}{9}=\frac{5}{9} \end{aligned} $

$ \therefore \quad P(A^{\prime} \cap B^{\prime})=P(A^{\prime}) \cdot P(B^{\prime})=\frac{2}{5} \cdot \frac{5}{9}=\frac{2}{9} $

Hence, the correct option is $(d)$.

70. If two events are independent, then

(a) they must be mutually exclusive

(b) the sum of their probabilities must be equal to 1

(c) (a) and (b) both are correct

(d) none of the above is correct

Show Answer

Solution

For independent events $A$ and $B, P(A) \cdot P(B)=P(A \cap B)$

So, they will not be mutually exclusive.

If $P(A)+P(B)=1$, they are exhaustive events and for independent events $A$ and $P(A \cap B) \neq 0$.

Hence, the correct option is $(d)$.

71. Let $A$ and $B$ be two events such that $P(A)=\frac{3}{8}, P(B)=\frac{5}{8}$ and $P(A \cup B)=\frac{3}{4}$. Then $P(A / B) \cdot P(A^{\prime} / B)$ is equal to

(a) $\frac{2}{5}$

(b) $\frac{3}{8}$

(c) $\frac{3}{20}$

(d) $\frac{6}{25}$

Show Answer

Solution

Given that: $P(A)=\frac{3}{8}, P(B)=\frac{5}{8}$ and $P(A \cup B)=\frac{3}{4}$

$ \begin{matrix} \therefore & P(A \cup B) & =P(A)+P(B)-P(A \cap B) \\ \frac{3}{4} & =\frac{3}{8}+\frac{5}{8}-P(A \cap B) \\ \Rightarrow \quad P(A \cap B) & =\frac{3}{8}+\frac{5}{8}-\frac{3}{4}=\frac{1}{4} \end{matrix} $

$ \begin{aligned} \text{ Now } P(A / B) \cdot P(A^{\prime} / B) & =\frac{P(A \cap B)}{P(B)} \cdot \frac{P(A^{\prime} \cap B)}{P(B)} \\ & =\frac{P(A \cap B)}{P(B)} \cdot \frac{P(B)-P(A \cap B)}{P(B)} \\ & =\frac{\frac{1}{5}}{\frac{5}{8}} \cdot \frac{(\frac{5}{8}-\frac{1}{4})}{\frac{5}{8}}=\frac{2}{5} \cdot \frac{3}{5}=\frac{6}{25} \end{aligned} $

Hence, the correct option is $(d)$.

72. If the events $A$ and $B$ are independent, then $P(A \cap B)$ is equal to

(a) $P(A)+P(B)$

(b) $P(A)-P(B)$

(c) $P(A) \cdot P(B)$

(d) $\frac{P(A)}{P(B)}$

Show Answer

Solution

Since $A$ and $B$ are two independent events

$ \therefore \quad P(A \cap B)=P(A) \cdot P(B) $

Hence, the correct option is (c).

73. Two events $E$ and $F$ are independent. If $P(E)=0.3$ and $P(E \cup F)=0.5$, then $P(E / F)-P(F / E)$ equals

(a) $\frac{2}{7}$

(b) $\frac{3}{35}$

(c) $\frac{1}{70}$

(d) $\frac{1}{7}$

Show Answer

Solution

Given that: $E$ and $F$ are independent events such that $P(E)=0.3$ and $P(E \cup F)=0.5$

$ \begin{matrix} P(E \cup F) =P(E)+P(F)-P(E \cap F) \\ 0.5 =0.3+P(F)-P(E) \cdot P(F) \\ \Rightarrow \quad 0.5-0.3 =P(F)[1-P(E)] \Rightarrow 0.2=P(F)(1-0.3) \\ \Rightarrow \quad 0.2 =P(F) \cdot(0.7) \\ \therefore \quad P(F) =\frac{0.2}{0.7}=\frac{2}{7} \\ \\ \text{ Now } P(E / F)-P(F / E) =\frac{P(E \cap F)}{P(F)}-\frac{P(E \cap F)}{P(E)} \\ =\frac{P(E) \cdot P(F)}{P(F)}-\frac{P(E) \cdot P(F)}{P(E)} =P(E)-P(F)=\frac{3}{10}-\frac{2}{7}=\frac{1}{70} \end{matrix} $

Hence, the correct option is (c).

74. A bag contains 5 red and 3 blue balls. If 3 balls are drawn at random without replacement, then the probability of getting exactly one red ball is

(a) $\frac{45}{196}$

(b) $\frac{135}{392}$

(c) $\frac{15}{56}$

(d) $\frac{15}{29}$

Show Answer

Solution

Given that: Bag contains 5 red and 3 blue balls.

Probability of getting exactly one red ball if 3 balls are randomly drawn without replacement

$=P(R) \cdot P(B) \cdot P(B)+P(B) \cdot P(R) \cdot P(B)+P(B) \cdot P(B) \cdot P(R)$

$=\frac{5}{8} \cdot \frac{3}{7} \cdot \frac{2}{6}+\frac{3}{8} \cdot \frac{5}{7} \cdot \frac{2}{6}+\frac{3}{8} \cdot \frac{2}{7} \cdot \frac{5}{6}=\frac{30}{336}+\frac{30}{336}+\frac{30}{336}=\frac{90}{336}=\frac{15}{56}$

Hence, the correct option is (c).

75. Refer to Question 74 above. The probability that exactly two of the three balls were red, the first ball being red is

(a) $\frac{1}{3}$

(b) $\frac{4}{7}$

(c) $\frac{15}{28}$

(d) $\frac{5}{28}$

Show Answer

Solution

According to Question 74,

Let $E_1$ be the event that first ball is red.

$E_2$ be the event that exactly two of the three balls are red.

$\therefore \quad P(E_1)=P(R) \cdot P(R) \cdot P(B)+P(R) \cdot P(R) \cdot P(R)+P(R) \cdot P(B) \cdot P(R)$ $+P(R) \cdot P(B) \cdot P(B)$

$=\frac{5}{8} \cdot \frac{4}{7} \cdot \frac{3}{6}+\frac{5}{8} \cdot \frac{4}{7} \cdot \frac{3}{6}+\frac{5}{8} \cdot \frac{3}{7} \cdot \frac{4}{6}+\frac{5}{8} \cdot \frac{3}{7} \cdot \frac{2}{6}$

$=\frac{60}{336}+\frac{60}{336}+\frac{60}{336}+\frac{30}{336}=\frac{210}{336}$

$P(E_1 \cap E_2)=P(R) \cdot P(B) \cdot P(R)+P(R) \cdot P(R) \cdot P(B)$

$=\frac{5}{8} \cdot \frac{3}{7} \cdot \frac{4}{6}+\frac{5}{8} \cdot \frac{4}{7} \cdot \frac{3}{6}=\frac{60}{336}+\frac{60}{336}=\frac{120}{336}$

$\therefore P(E_2 / E_1)=\frac{P(E_1 \cap E_2)}{P(E_1)}=\frac{120 / 336}{210 / 336}=\frac{4}{7}$

Hence, the correct option is $(b)$.

76. Three persons A, B and C fire at a target in turn, starting with A. Their probabilities of hitting the target are $0.4,0.3$ and 0.2 respectively. The probabilities of two hits is

(a) 0.024

(b) 0.188

(c) 0.336

(d) 0.452

Show Answer

Solution

Given that: $P(A)=0.4, P(B)=0.3$ and $P(C)=0.2$

Also $P(\overline{A})=1-0.4=0.6, P(\overline{B})=1-0.3=0.7$

and $P(\bar{{}C})=1-0.2=0.8$

$\therefore$ Probabilities of two hits

$=P(A) \cdot P(B) \cdot P(\overline{C})+P(A) \cdot P(\overline{B}) \cdot P(C)+P(\overline{A}) \cdot P(B) \cdot P(C)$

$=0.4 \times 0.3 \times 0.8+0.4 \times 0.7 \times 0.2+0.6 \times 0.3 \times 0.2$

$=0.096+0.056+0.036=0.188$

Hence, the correct option is $(b)$.

77. Assume that in a family, each child is equally likely to be a boy or a girl. A family with three children is chosen at random. The probability that the eldest child is a girl given that the family has atleast one girl is

(a) $\frac{1}{2}$

(b) $\frac{1}{3}$

(c) $\frac{2}{3}$

(d) $\frac{4}{7}$

Show Answer

Solution

Let $G$ denotes the girl and $B$ denotes the boy of the given family.

$\text{So}n(S)=\{(BGG),(GBG),(GGB),(GBB),(BGB),(BBG),(BBB), (GGG)\}$

Let $E_1$ be the event that the family has alteast one girl.

$\therefore E_1=\{(BGG),(GBG),(GGB),(GBB),(BGB),(BBG),(GGG)\}$

Let $E_2$ be the event that the eldest child is a girl.

$ \begin{aligned} & \therefore \quad E_2={(GBG),(GGB),(GBB),(GGG)} \\ & (E_1 \cap E_2)={(GBB),(GGB),(GBG),(GGG)} \\ & \therefore P(E_2 / E_1)=\frac{P(E_1 \cap E_2)}{P(E_1)}=\frac{4 / 8}{7 / 8}=\frac{4}{7} \end{aligned} $

Hence, the correct option is $(d)$.

78. If a die is thrown and a card is selected at random from a deck of 52 playing cards. The probability of getting an even number on the die and a spade card is

(a) $\frac{1}{2}$

(b) $\frac{1}{4}$

(c) $\frac{1}{8}$

(d) $\frac{3}{4}$

Show Answer

Solution

Let $E_1$ be the event of getting even number on the die. $E_2$ be the event of selecting a spade card.

$\therefore P(E_1)=\frac{3}{6}=\frac{1}{2}$ and $P(E_2)=\frac{13}{52}=\frac{1}{4}$

So $\quad P(E_1 \cap E_2)=P(E_1) \cdot P(E_2)=\frac{1}{2} \cdot \frac{1}{4}=\frac{1}{8}$

Hence, the correction option is (c).

79. A box contains 3 orange balls, 3 green balls and 2 blue balls. Three balls are drawn at random from the box without replacement. The probability of drawing 2 green balls and one blue ball is

(a) $\frac{3}{28}$

(b) $\frac{2}{21}$

(c) $\frac{1}{28}$

(d) $\frac{167}{168}$

Show Answer

Solution

Probability of drawing 2 green and 1 blue balls $=P(G) \cdot P(G) \cdot P(B)+P(G) \cdot P(B) \cdot P(G)+P(B) \cdot P(G) \cdot P(G)$

$=\frac{3}{8} \cdot \frac{2}{7} \cdot \frac{2}{6}+\frac{3}{8} \cdot \frac{2}{7} \cdot \frac{2}{6}+\frac{2}{8} \cdot \frac{3}{7} \cdot \frac{2}{6}=\frac{12}{336}+\frac{12}{336}+\frac{12}{336}=\frac{36}{336}=\frac{3}{28}$

Hence, the correct option is $(a)$.

80. A flashlight has 8 batteries out of which 3 are dead. If two batteries are selected without replacement and tested then the probability that both are dead is

(a) $\frac{33}{56}$

(b) $\frac{9}{64}$

(c) $\frac{1}{14}$

(d) $\frac{3}{28}$

Show Answer

Solution

Required probability $=P($ dead $) \cdot P($ dead $)$

$ =\frac{3}{8} \cdot \frac{2}{7}=\frac{3}{28} $

Hence, the correct option is $(d)$.

81. If eight coins are tossed together, then the probability of getting exactly 3 heads is

(a) $\frac{1}{256}$

(b) $\frac{7}{32}$

(c) $\frac{5}{32}$

(d) $\frac{3}{32}$

Show Answer

Solution

Here, $n=8, p=\frac{1}{2}, q=1-\frac{1}{2}=\frac{1}{2}$ and $r=3$

We know that $P(x=r)={ }^{n} C_r p^{r} . q^{n-r}$

$ \begin{aligned} \therefore \quad P(x=3) & ={ }^{8} C_3(\frac{1}{2})^{3}(\frac{1}{2})^{8-3} \\ & =\frac{8 !}{3 ! \cdot 5 !} \cdot(\frac{1}{2})^{3}(\frac{1}{2})^{5}=56 \cdot(\frac{1}{2})^{8}=56 \cdot \frac{1}{256}=\frac{7}{32} \end{aligned} $

Hence, the correct option is $(b)$.

82. Two dice are thrown. If it is known that the sum of numbers on the dice was less than 6 , the probability of getting a sum 3 , is

(a) $\frac{1}{18}$

(b) $\frac{5}{18}$

(c) $\frac{1}{5}$

(d) $\frac{2}{5}$

Show Answer

Solution

Let $E_1$ be the event showing the sum of the numbers on the two dice was less than 6 and $E_2$ be the event that the sum of the numbers is 3 .

$ \begin{aligned} & \therefore \quad E_1=\{(1,1),(1,2),(2,1),(1,3),(3,1),(1,4),(4,1), \\ &(2,2),(2,3),(3,2)\} \\ & \text{ and } \quad n(E_1)=10 \\ & E_2={(1,2),(2,1)} \Rightarrow n(E_2)=2 \text{ and } n(E_1 \cap E_2)=2 \end{aligned} $

$\therefore$ Required probability

$ P(E_2 / E_1)=\frac{n(E_1 \cap E_2)}{n(E_1)}=\frac{2}{10}=\frac{1}{5} $

Hence, the correct option is (c).

83. Which one is not a requirement of a Binomial distribution?

(a) There are two outcomes for each trial. (b) There is a fixed number of trials.

(c) The outcomes must be dependent on each other.

(d) The probability of success must be the same for all trials.

Show Answer

Solution

We know that for a Binomial distribution, the outcomes must not be dependent on each other.

Hence, the correct option is (c).

84. If two cards are drawn from a well shuffled deck of 52 playing cards with replacement, then the probability that both cards are queens is

(a) $\frac{1}{13} \times \frac{1}{13}$

(b) $\frac{1}{13}+\frac{1}{13}$

(c) $\frac{1}{13} \times \frac{1}{17}$

(d) $\frac{1}{13} \times \frac{4}{51}$

Show Answer

Solution

Probability of getting Queen $=\frac{4}{52}$

So, the required probability

$ \begin{aligned} & =P(\text{ Queen }) \cdot P(\text{ Queen }) \\ & =\frac{4}{52} \cdot \frac{4}{52}=\frac{1}{13} \cdot \frac{1}{13} \quad \text{ (with replacement) } \end{aligned} $

Hence, the correct option is $(a)$.

85. The probability of guessing correctly atleast 8 out of 10 answers on a true false type examination is

(a) $\frac{7}{64}$

(b) $\frac{7}{128}$

(c) $\frac{45}{1024}$

(d) $\frac{7}{41}$

Show Answer

Solution

Here, $n=10, p=\frac{1}{2}$ and $q=\frac{1}{2} \quad$ (for true/false questions) and $r \geq 8$ i.e. $8,9,10$

$ \begin{aligned} \therefore P(X \geq 8) & =P(x=8)+P(x=9)+P(x=10) \\ & ={ }^{10} C_8(\frac{1}{2})^{8}(\frac{1}{2})^{2}+{ }^{10} C_9(\frac{1}{2})^{9}(\frac{1}{2})+{ }^{10} C _{10}(\frac{1}{2})^{10}(\frac{1}{2})^{0} \\ & =45 \cdot(\frac{1}{2})^{10}+10 \cdot(\frac{1}{2})^{10}+(\frac{1}{2})^{10}=(\frac{1}{2})^{10}(45+10+1) \\ & =56 \times \frac{1}{1024}=\frac{7}{128} \end{aligned} $

Hence, the correct option is (b).

86. The probability that a person is not a swimmer is 0.3 . The probability that out of 5 persons 4 are swimmers is

(a) ${ }^{5} C_4(0.7)^{4}(0.3)$

(b) ${ }^{5} C_1(0.7)(0.3)^{4}$

(c) ${ }^{5} C_4(0.7)(0.3)^{4}$

(d) $(0.7)^{4}(0.3)$

Show Answer

Solution

Given that: $\overline{P}=0.3 \therefore p=0.7$ and $q=1-0.7=0.3$ $n=5$ and $r=4$

We know that

$ \begin{aligned} & P(X=r)={ }^{n} C_r(p)^{r} \cdot(q)^{n-r} \\ \therefore \quad & P(x=4)={ }^{5} C_4(0.7)^{4}(0.3)^{5-4}={ }^{5} C_4(0.7)^{4}(0.3) \end{aligned} $

Hence, the correct option is (a).

87. The probability distribution of a discrete random variable $X$ is given below:

$\begin{array}{|l|l|l|l|l|} \hline X & 2 & 3 & 4 & 5 \\ \hline P(X) & \frac{5}{k} & \frac{7}{k} & \frac{9}{k} & \frac{11}{k} \\ \hline \end{array}$

The value of $k$ is

(a) 8

(b) 16

(c) 32

(d) 48

Show Answer

Solution

We know that $\sum _{i=1}^{n} P(X_i)=1$

$ \begin{aligned} \therefore \quad \frac{5}{k}+\frac{7}{k}+\frac{9}{k}+\frac{11}{k} & =1 \\ \frac{32}{k} & =1 \Rightarrow k=32 . \end{aligned} $

Hence, the correct option is (c).

88. For the following probability distribution:

$\begin{array}{|l|l|l|l|l|l|} \hline X & -4 & -3 & -2 & -1 & 0\\ \hline P(X) & 0.1 & 0.2 & 0.3 & 0.2 & 0.2 \\ \hline \end{array}$

$E(X)$ is equal to:

(a) 0

(b) -1

(c) -2

(d) -1.8

Show Answer

Solution

We know that:

$ \begin{aligned} E(X) & =\sum _{i=1}^{n} X_i P_i \\ & =(-4)(0.1)+(-3)(0.2)+(-2)(0.3)+(-1)(0.2)+0(0.2) \\ & =-0.4-0.6-0.6-0.2=-1.8 \end{aligned} $

Hence, the correct option is $(d)$.

89. For the following probability distribution

$\begin{array}{|l|l|l|l|l|l|} \hline X & 1 & 2 & 3 & 4 \\ \hline P(X) & \frac{1}{10} & \frac{3}{10} & \frac{3}{10} & \frac{2}{10} \\ \hline \end{array}$

$E(X^{2})$ is equal to

(a) 3

(b) 5

(c) 7

(d) 10

Show Answer

Solution

We know that

$ \begin{aligned} E(X^{2}) & =\sum _{i=1}^{n} P_i X_i^{2} \\ & =1 \times \frac{1}{10}+4 \times \frac{1}{5}+9 \times \frac{3}{10}+16 \times \frac{2}{5} \end{aligned} $

$ =\frac{1}{10}+\frac{4}{5}+\frac{27}{10}+\frac{32}{5}=\frac{28}{10}+\frac{36}{5}=\frac{100}{10}=10 $

Hence, the correct option is (d).

90. Suppose a random variable $X$ follows the Binomial distribution with parameters $n$ and $p$, where $0<p<1$. If $\frac{P(x=r)}{P(x=n-r)}$ is independent of $n$ and $r$, then $p$ equals

(a) $\frac{1}{2}$

(b) $\frac{1}{3}$

(c) $\frac{1}{5}$

(d) $\frac{1}{7}$

Show Answer

Solution

$\quad P(X=r)={ }^{n} C_r p^{r} q^{n-r}=\frac{n !}{r !(n-r) !} p^{r} \cdot(1-p)^{n-r}$

$ \begin{aligned} P(X=n-r) & ={ }^{n} C _{n-r} p^{n-r} \cdot(q)^{n-(n-r)}={ }^{n} C _{n-r} p^{n-r} \cdot q^{r} \\ & =\frac{n !}{(n-r) !(n-n+r) !} p^{n-r} q^{r}=\frac{n !}{(n-r) ! r !} \cdot p^{n-r} \cdot q^{r} \end{aligned} $

Now $\frac{P(x=r)}{P(x=n-r)}=\frac{\frac{n !}{r !(n-r) !} \cdot p^{r} \cdot(1-p)^{n-r}}{\frac{n !}{r !(n-r) !} \cdot p^{n-r} \cdot(1-p)^{r}}=\frac{(\frac{1-p}{p})^{n-r}}{(\frac{1-p}{p})^{r}}$

The above expression will be independent of $n$ and $r$ if

$ (\frac{1-p}{p})=1 \Rightarrow \frac{1}{p}=2 \Rightarrow p=\frac{1}{2} $

Hence, the correct option is (a).

91. In a college, $30 %$ students fail in Physics, $25 %$ fail in Mathematics and $10 %$ fail in both. One student is chosen at random. The probability that she fails in Physics if she has failed in Mathematics is

(a) $\frac{1}{10}$

(b) $\frac{2}{5}$

(c) $\frac{9}{20}$

(d) $\frac{1}{3}$

Show Answer

Solution

Let $E_1$ be the event that the student fails in Physics and $E_2$ be the event that she fails in Mathematics.

$\therefore \quad P(E_1)=\frac{30}{100}, P(E_2)=\frac{25}{100}$ and $P(E_1 \cap E_2)=\frac{10}{100}$

$\therefore P(E_1 / E_2)=\frac{P(E_1 \cap E_2)}{P(E_2)}=\frac{10 / 100}{25 / 100}=\frac{2}{5}$

Hence, the correct option is (b).

92. A and B are two students. Their chances of solving a problem correctly are $\frac{1}{3}$ and $\frac{1}{4}$ respectively. If the probability of their making a common error is $\frac{1}{20}$ and they obtain the same answer, then the probability of their answer to be correct is

(a) $\frac{1}{12}$

(b) $\frac{1}{40}$

(c) $\frac{13}{120}$

(d) $\frac{10}{13}$

Show Answer

Solution

Let $E_1$ be the event that both of them solve the problem.

$\therefore \quad P(E_1)=\frac{1}{3} \times \frac{1}{4}=\frac{1}{12}$

and $E_2$ be the event that both of them same incorrectly the problem.

$\therefore \quad P(E_2)=(1-\frac{1}{3}) \times(1-\frac{1}{4})=\frac{2}{3} \times \frac{3}{4}=\frac{1}{2}$

Let $H$ be the event that both of them get the same answer.

Here, $P(H / E_1)=1, P(H / E_2)=\frac{1}{20}$

$\therefore P(E_1 / H)=\frac{P(E_1) \cdot P(H / E_1)}{P(E_1) \cdot P(H / E_1)+P(E_2) \cdot P(H / E_2)}$

$=\frac{\frac{1}{12} \times 1}{\frac{1}{12} \times 1+\frac{1}{2} \times \frac{1}{20}}=\frac{\frac{1}{12}}{\frac{1}{12}+\frac{1}{40}}=\frac{\frac{1}{12}}{\frac{10+3}{120}}=\frac{1 / 12}{13 / 120}=\frac{10}{13}$

Hence, the correct option is (d).

93. A box has 100 pens of which 10 are defective. What is the probability that out of a sample of 5 pens drawn one by one with replacement atmost one is defective?

(a) $(\frac{9}{10})^{5}$

(b) $\frac{1}{2}(\frac{9}{10})^{4}$

(c) $\frac{1}{2}(\frac{9}{10})^{5}$

(d) $(\frac{9}{10})^{5}+\frac{1}{2}(\frac{9}{10})^{4}$

Show Answer

Solution

Here, $n=5, p=\frac{10}{100}=\frac{1}{10}$ and $q=1-\frac{1}{10}=\frac{9}{10}$ and $r \leq 1$

We know that

$ \text{ So } \quad \begin{aligned} P(X=r) & ={ }^{n} C_r p^{r} q^{n-r} \\ P(x \leq 1) & =P(x=0)+P(x=1) \\ & ={ }^{5} C_0(\frac{1}{10})^{0}(\frac{9}{10})^{5}+{ }^{5} C_1(\frac{1}{10})(\frac{9}{10})^{4} \end{aligned} $

$ =(\frac{9}{10})^{5}+\frac{5}{10}(\frac{9}{10})^{4}=(\frac{9}{10})^{5}+\frac{1}{2}(\frac{9}{10})^{4} $

Hence, the correct option is (d).

True/False

94. Let $P(A)>0$ and $P(B)>0$. Then $A$ and $B$ can be both mutually exclusive and independent.

Show Answer

Solution

False

95. If $A$ and $B$ are independent events, then $A^{\prime}$ and $B^{\prime}$ are also independent.

Show Answer

Solution

True

96. If A and B are mutually exclusive events, then they will be independent also.

Show Answer

Solution

False

97. Two independent events are always mutually exclusive.

Show Answer

Solution

False

98. If $A$ and $B$ are two independent events, then

$ P(A \text{ and } B)=P(A) \cdot P(B) \text{. } $

Show Answer

Solution

True

99. Another name for the mean of a probability distribution is expected value.

Show Answer

Solution

True $\qquad [\because E(X)=\sum X_i P(X_i)]$

100. If $A$ and $B^{\prime}$ are independent events, then $P(A^{\prime} \cup B)=1-P(A) \cdot P(B^{\prime})$

Show Answer

Solution

True $\quad[\because P(A^{\prime} \cup B)=1-P(A \cap B^{\prime})=1-P(A) \cdot P(B^{\prime})]$

101. If $A$ and $B$ are independent events, then

$P($ exactly one of $A, B$ occurs $)=P(A) \cdot P(B^{\prime})+P(B) \cdot P(A^{\prime})$

Show Answer

Solution

True

102. If $A$ and $B$ are two events such that $P(A)>0$ and $P(A)+P(B)>1$,

then $P(B / A) \geq 1-\frac{P(B^{\prime})}{P(A)}$

Show Answer

Solution

False

$ \begin{matrix} {[\because P(B / A)=\frac{P(A \cap B)}{P(A)}=\frac{P(A)+P(B)-P(A \cup B)}{P(A)}}>\frac{1-P(A \cup B)}{P(A)}] \end{matrix} $

103. If $A, B$ and $C$ are three independent events such that

$P(A)=P(B)=P(C)=p$,

then P (atleast two of A, B and C occur) $=3 p^{2}-2 p^{3}$

Show Answer

Solution

True

Since $P$ (atleast two of $A, B$ and $C$ occur)

$ \begin{aligned} & =p \times p \times(1-p)+(1-p) \cdot p \cdot p+p(1-p) \cdot p+p \cdot p \cdot p \\ & =3 p^{2}(1-p)+p^{3}=3 p^{2}-3 p^{3}+p^{3}=3 p^{2}-2 p^{3} \end{aligned} $

Fillers

104. If $A$ and $B$ are two events such that $P(A / B)=p, P(A)=p$,

$ P(B)=\frac{1}{3} \text{ and } P(A \cup B)=\frac{5}{9} \text{, then } p \text{ is equal to } $ ……

Show Answer

Solution

Given that: $P(A)=p, P(B)=\frac{1}{3}$ and $P(A \cup B)=\frac{5}{9}$

$ \begin{aligned} P(A / B) & =\frac{P(A \cap B)}{P(B)}=p \Rightarrow P(A \cap B)=p \cdot P(B)=p \cdot \frac{1}{3} \\ \text{ and } P(A \cup B) & =P(A)+P(B)-P(A \cap B) \\ \frac{5}{9}=p+\frac{1}{3} & -\frac{p}{3} \Rightarrow \frac{5}{9}-\frac{1}{3}=\frac{2 p}{3} \Rightarrow \frac{2}{9}=\frac{2 p}{3} \Rightarrow p=\frac{1}{3} \end{aligned} $

Hence, $p$ is equal to $\frac{1}{3}$.

105. If $A$ and $B$ are such that $P(A^{\prime} \cup B^{\prime})=\frac{2}{3}$ and $P(A \cup B)=\frac{5}{9}$, then $P(A^{\prime})+P(B^{\prime})$ is equal to ……

Show Answer

Solution

Here,

$ P(A^{\prime} \cup B^{\prime})=\frac{2}{3} \text{ and } P(A \cup B)=\frac{5}{9} $

$ \begin{aligned} & \therefore & 1-P(A \cap B) & =\frac{2}{3} \\ & \Rightarrow & P(A \cap B) & =1-\frac{2}{3}=\frac{1}{3} \end{aligned} $

Now $\quad P(A^{\prime})+P(B^{\prime})=1-P(A)+1-P(B)=2-[P(A)+P(B)]$

$ \begin{aligned} & =2-[P(A \cup B)+P(A \cap B)] \\ & =2-[\frac{5}{9}+\frac{1}{3}]=2-\frac{8}{9}=\frac{10}{9} \end{aligned} $

Hence, the value of the filler is $\frac{10}{9}$.

106. If $X$ follows Binomial distribution with parameters $n=5, p$ and $P(X=2)=9 P(X=3)$, then $p$ is equal to ……

Show Answer

Solution

Given that: $P(X=2)=9 P(X=3)$

$ \Rightarrow{ }^{5} C_2 p^{2} q^{3}=9 .{ }^{5} C_3 p^{3} q^{2} $

$ \begin{matrix} \Rightarrow \frac{1}{9} =\frac{{ }^{5} C_3 p^{3} q^{2}}{{ }^{5} C_2 p^{2} q^{3}} \Rightarrow \frac{1}{9}=\frac{p}{q} \quad[\because{ }^{5} C_3={ }^{5} C_2] \\ \Rightarrow 9 p =q \\ \Rightarrow 9 p =1-p \Rightarrow 9 p+p=1 \Rightarrow 10 p=1 \therefore p=\frac{1}{10} \end{matrix} $

Hence, the value of the filler is $\frac{1}{10}$.

107. Let $X$ be a random variable taking values $x_1, x_2, x_3, \ldots, x_n$ with probabilities $p_1, p_2, p_3, \ldots, p_n$ respectively. Then $Var(X)$ is equal to ……

Show Answer

Solution

$\quad Var(X)=E(X^{2})-[E(X)]^{2}$

$=\sum X^{2} P(X)-[\sum X . P(X)]^{2}=\sum p_i x_i^{2}-(\sum p_i x_i)^{2}$

Hence, $Var(X)$ is equal to $\sum p_i x_i^{2}-(\sum p_i x_i)^{2}$.

108. Let $A$ and $B$ be two events. If $P(A / B)=P(A)$, then $A$ is of B ……

Show Answer

Solution

$ \begin{matrix} \because \qquad P(A / B) =\frac{P(A \cap B)}{P(B)} \\ \Rightarrow P(A) =\frac{P(A \cap B)}{P(B)} \\ \Rightarrow P(A \cap B) =P(A) \cdot P(B) \end{matrix} $

So, $A$ is independent of $B$.



Table of Contents